Download as pdf or txt
Download as pdf or txt
You are on page 1of 488

‫ ﻣﺎرس‬١ ‫ﻣﻨﺎﻗﺸﺔ‬

✅✅
21. Multiple myloma : malignant tumour of plasma cells
22. Clinical case about removable partial denture with acrylic base with low

✅✅
interocclusal space at rest and bruxism what type of teeth to use:
Hi impact acrylic
Acetyl acrylic
Nano filled acrylic
Porcelain

23. -Patient with low muscle activity and have interference in removable denture
how to correct:
Intraoral modification with selective grinding?

✅✅
Intraoral modification with path generated path
Extraoral remount and sent to laboratory

24. The patient has generalized intrinsic staining . which material


would you use to treat this case ?
A_alumina

✅✅
B_cast ceramics /
C- etched porcelain
✅✅
25. Pic of flap unaesthetic :
A-free gingival flap
B- semilunar
C-Double papilla
D-connective tissue flap

26. Patient with ectodermal dysplasia, with multiple missing teeth. planning for
fixed removable appliance. what is the sequence of treatment?
Deliver a fixed appliance first then removable.

✅✅
Deliver both together.
Deliver removable appliances first then fixed.

✳✳27. The posterior palatal bar should be placed in a relationship to

✅✅
incorporate the hamular notch on each side
which is anterior to the junction of the movable and immovable soft palate
which is posterior to the junction of the movable and immovable soft palate
extending from one hamular notch to the other hamular notch across the fovea
palatine

28. The diameter of dentinal tubules AT pulp region ?


A_0. 5 micron

✅✅
B_1 micron
C_2-3 um

29. A patient of yours walks into your office with the following complaint. "When I
smile, my upper denture does not hold." Which area of the denture base needs

✅✅
to be adjusted?
A• Labial notch and labial flange
B• Buccal notch and buccal flange
C• Posterior border
D• Distobuccal flange
✳✳30. pemphigus valgaris affect more:
A-Lingual mucosa
B-proximal
C-Buccal mucosa✳✳
D_floor of mouth
31. Space between marginal ridge of molar class1 cavity:
A-1 mm
B-1.5mm > premolar
C-2mm

✳✳32. very destructive “vital” molar and you will do a direct temporary-crown
what material you should avoid?
A_Methylethylmetha acrylate
B_methyl acrylate, > mostly used in clinics > irritant pulp
C_methylpolyvenile acrylate,

✅✅
D_bis composite
polymethylmethacrylate
✅✅
33. -Antero-posterior opening of Antero-posterior palatal strap
major connector (15mm)
34. In class 2 removable partial denture the best location for indirect retention
A. as Anterior as possible in the same side of the edentulous area

✅✅
B. as anterior as possible in the opposite side of the edentulous area
C. posterior as possible in the other area
D. in the canine of same area of edentulous area
25. Child came to the clinic with amalgam restoration fracture at isthmus portion ,
this fracture due to ?
A_Wide prep at isthmus

✅✅
B_High occlusal
C_Shallow prep > it should be equal parallel enough
D_Constricted isthmus
‫‪Dr.Afnan‬‬
‫اذﻛﺮوﻧﻲ ﺑﺪﻋﻮة‬
‫ مارس‬٢ ‫مناقشة‬

1. How does internal resorption appear in radiograph?


‫نسبه نجاح الزرعه مع سمنتد كراون‪‎2-‬‬
✅✅
‎ -‫صوره لي ويليام بروب ويقول ايش هو‬
3
William probe pic

Combination instruments for probing caries and parodontium. Combination


instruments for probingcaries and parodontium. Most explorers and periodontal
probes are available also in single-ended designs
➡️internal resorption✅✅
4-obtura 2?
thermoplastic injection technique

**🌸Thermomechnical compaction techniques 👉🏻Mcspadden.


🌸Thermoplastic injection Technique 👉🏻Obtura System or Optera III
🌸Continuous wave obturaution +warm vertical compaction 👉🏻both we can use
*system B plugger*.
. how to fill the internal resorption?
Lateral compaction

✅✅
heat master cone then lateral compaction

👆🏾👆🏾
thermoplastisized

‫صيغه سوال مشابهه‬

5. classification of ANUG?

✅✅
‫سأل عن المارجنال جنجفا وقال دا اي كالس !؟‬
Class 3
✅✅
6. pt taken prednisolone 10mg and need simple extraction?
No need to double dose (simple)

➡️➡️ ‫نقص من الغدة وعدم فرز للكورتيزون‬


** primary

Secondry ➡️➡️ ‫ضعف مستقبالت الجسم للتعرف ع الكورتيزون‬

7-‫مريض مستأصل جزء من الليفر‬


‫السؤال االول نوع االناسيزيا‪‎‬‬

‫✅✅‬
‫‪‎‬‬
‫تقريبا‬ ‫??والسوال التاني ايش حتعمل قبل الخلع‬

‫✅✅‬
‫‪Ester if they not mentioned then articaine‬‬
‫‪PT test‬‬
✅✅
8. ‫اسئله‬
‎ ‫شوجرن سيندروم جاب عنها‬

✅✅
Dry mouth dry eye

✅✅
Associated with rheumatoid arthritis

✅✅
Sexondary type associated with rumatoid arthritis
Schimers test less than 5 drops in 5 min
Tested by antiRo antiLa antibodies ✅✅
9. chlxr chips for anterior teeth and ask what is this?
✅✅✅ 💡
10. Pic angina bullosa hemorrhagica?
Angina Bullosa Hemorrhagica

is a rare acute and benign

🔹
blood blistering oral disorder.

🔸
Etiology The exact etiology is unknown.
. However, mild trauma and
the chronic use of steroid inhalers seemto play an important role in the
development of the lesions.
‫االثيكس ‪11.‬‬
‫‪‎‬‬ ‫كم سؤال عن‬
‫‪‎‬‬
‫عندو‬ ‫والمريض اللي يتحول عند دكتور ويسالو يكمل العالج‬
‫‪‎‬‬
‫المعلم‬ ‫والطفل اللي بالمدرسه وناخد الكونسنت من‬
‫‪‎‬‬
‫الوالدين‬ ‫والمريض عمرو ‪ ١٦‬يبغى يسوي دايماند من غير موافقه‬

‫✅✅‬
‫?‪12. virus of AIDs‬‬
‫‪HIV‬‬
✅✅
13. ‎isolated abutment?
Pier abutment
14. type of occlusion

✅✅
In class one opposite by complete denture??
Bilateral Balance occlusion
‫مريض عاوز يسوي ملتبل ريستوريشن‬
‫ايش نوع االوكلوشن !؟‬
✅✅
‫سنتريك ريليشن‬
‫سنتريك اكولوشن‬
....
.....

15. Questions about lichen planus


Picture?

✅✅
16. ‫الخيارات‬
Rpi
Rpa
Revers
Aker
‫الفرق بينهم🏾👇🏾👇‬
‫‪ .17‬بقول سبيس انفكشن رايح على السب ماندبل والسب لنجوال وماقادر يتنفس‬
‫سييناريو طويل وانها‬
‫‪Involve‬‬
‫‪Sub mental, Sub mandible and sub lingual‬‬

‫‪Answer: ludwig angina‬‬


‫✅✅✅‬

‫الريسيبروكال؟ ‪18.‬‬
‫‪‎‬‬ ‫االبتمنت يتحرك ليش الخيارات كلها عن‬
‫‪:‬مريض عندو كومبليت دنشر ايش الشي اللي ما يقدر يسويه ‪19.‬‬
Soft food
Hard food

✅✅
Creamy icecream
Cuuting on canine area
‫فالس ‪20.‬‬
‫‪‎‬‬ ‫?كم سوال عن البوكت هو انفرا بوني وال سوبرا بوني وال ترو وال‬

‫✅✅‬
‫‪False or pseudo pocket is when there is gingival swelling there will be increase in‬‬
‫‪poket depth without attachment loss‬‬

‫‪‎‬‬

‫سال عن البروفيالكتيك انتيبيوتك للبيدو ‪21.‬‬


‫الخيارات كانت‬
‫امبيسلين‬
‫اموكسسيلين‬
‫سيبروفالكساسين‬
‫الخيار الرابع ما متذكر‬
‫سوال عن االركتكشر بس جاب اسامي مختلفه‪22. -‬‬
‫وكدا ‪Inverted‬‬

‫ما ذكر بوزيتيف وال نجتف‬


‫جاي اسامي مختلفه‬
‫زي‬
‫‪Inverted‬‬
‫‪Converted‬‬
‫وكذا‬

‫‪Interdental bone inverted- negative‬‬ ‫✅✅‬


👆🏾👍🏾👍🏾‫جاات نفس الصورة دي‬
23. ‎pic about mandibular tori?

24. 7mm bucco-ling


And 7mm mesio distally what best diameter of implant (adjecent to tooth) ?

✅✅
3
-4
-5
‫✅✅‬
‫صوره لي هوريزنتال بون لوس بي نسبه ‪ ٢٠‬ونبغى نسوي امبالنت؟‪25. ‎‬‬
‫‪‎‬‬
‫نطلب‬ ‫‪ cbct‬مفروض‬
‫الجروث!؟ ‪26.‬‬
‫‪‎‬‬ ‫صورة االشعه عشان نحدد‬
‫‪‎‬‬
‫سيفالوتمرك‬ ‫اللي هي سيلاير‬
** the best and accurate ➡️➡️ hand and wrist radiograph✅✅
**Less than 17 year old ➡️➡️ cephalometric radiograph✅

Oldest pt. >17 y. ➡️➡️ hand and wrist radiograph.✅

Dr.Afnan
‫اذكروني بدعوة‬
‫ مارس‬٣ ‫مناقشة‬

27. Remaining dentin thickness is 2 mm what do ?


No need base or liner
Base
Liner

➡️ ✅✅
.....

➡️ ✅✅
Amalgam varnish
Conposite no need
28. Why chlorhexidine has a wide spectrum of antimicrobail proprties?

✅✅
A. Because of its cytotoxicity
B. Because of its substantivity
C. Because of its high concentration
D. Because of its tissue dissolving capacity

✅✅
29. High restoration on teeth with no bone loss and normal periodontal
Primary
or secondry trauma from occ?

Primary truma from occlusion =resultsfrom alteration in occlusal force high


filling

Secondary truma from occlusion = reduce ability of tissue to resist force

Primary trauma from occlusion: tissue reaction around a tooth with normal
periodontium
height

- Secondary trauma from occlusion : tissue reaction around a tooth with


reduced periodontium
heigh

30-50% bone loss = secondary trauma


30. Question about Himophilia A ?
Caused by factor viii

✅✅
If mild treated by desmopressin
If severe factor replacementor plasma

✅✅
31. Reed strenberg appearance in ?
Hodgkin lymphoma

✅✅✅✅✅
cHL is the most common type of Hodgkin lymphoma. About 95% of cases
of Hodgkin lymphoma are within the cHL category. cHL is diagnosed when
certain abnormal lymphocytes, known as Reed-Sternberg cells, are found

32. ‫اي صوره اشعه تحدد االمباكتد كناين‬


‫ وجاب‬cbct ‫لي امباكتد كنااين‬

✅✅✅✅
‫قال االمباكشن وين !؟‬
Cbct first choice
✅✅
?‫ جاب سؤال االوبليك ريدج في البرايمري بيربط بين اي كسب‬.33
Distoobuccal with mesiopalatal

The oblique ridge is found on the occlusal surfaces of maxillary molars. It is


formed by the union of the distal cusp ridge of the mesiolingual cusp and
the triangular ridgeof the distobuccal cusp. The oblique ridgesusually forms
the distal boundary of the central fossa
✅✅
34. Which cusp is absence in upper third molar!?
Distolingual
35. Question related to this picture. Management or staging 🤷‍♀️
36. Question about Amelogenesis imperfecta and dentinogenesis
imperfecta?

💡
dentinogenisis imperfecta features?

📍
Type 1.. DI..
occurs with osteogenesis imperfecta,
an inherited defect in collagen formation

🔆 results in osteoporotic brittle bones, bowing of the limbs, bitemporal


bossing, and blue sclera.

🔆 Primary teeth tend to be more


severely affected than permanent teeth.

🔆Periapical radiolucencies, bulbous crowns, obliteration of pulp


chambers, root fractures.

An amber translucent tooth color is common.

💡Type 2..DI..
🔆hereditary opalescent dentin
🔆Not associated with osteogenesis
🔆both primary and permanent dentitions
🔆bulbous crowns, obliteration of pulp chambers and
are equally affected

root fractures.

💡 type 3 is rare
📍bell-shaped crowns, especially in the permanen.
📍teeth with a shell-like appearance
and multiple pulp exposures
✅✅✅

✅✅
37. mobility of implant what meaning?
Failure
‫صوره ‪38.‬‬
‫‪‎‬‬ ‫سوال عن الليفورت جايب‬
✅✅
39. cement avoided in vital tooth ?
Poly methyl metha acrylate
✅✅
40. coupling agent of composite?
Metha acryloxy propyl trimetheaxy silane (GMMA -BPS)
composite contain = bis-gamma
coupling agent = 3 methacryloxypropyl trimethoxysilane (gamma-MPS)

41. bacteria of root caries?

✅✅
Abstract. Culture-based studies have shown that Streptococcus mutans
andlactobacilli are associated with root caries

42. disadvantage of gutta percha?


Shrinkage on cooling after heating
✅✅
43. tilted molar what clasp?
Ring clasp

Aesthatic + tilited + more stress + misolingual undercut = ring


Esthatic + mid buccal = Rpi
Undercut I bar
Mesiofacial undercut = worit wire
Rotation root = revers clasp
Class l + good oral hygiene = aker

✅✅
44. TB how much is remainin clinic?
4-6 hrs

HIV = 5-6 days


HBV = in room temp. 7days-6months
TB= 4-6 hrs

✅✅
45. ‫اول‬
‎ ‫?السؤال حق المريض عندو حشوه ونظافه لثه وخلع اي واحد فيهم نسوي‬
‫ حنبدأ اسكيلنق بعدين فيلنق بعدين خلع‬emergency ‫اذا مافي‬

✅✅
46. ‫معانا؟؟‬
‎ ‫لمن نخلع لور ايت كيف شكل الديفكت الحكون‬
Verical defect to distal second molar
✳️47. Resin bonded cement bridge fall what well you do
1.Remake
2.do prepration in lingual and re cement

✳️✅
3.send to lap
4- recementaion

✅✅
48. pt 60 age with scc‫?والولد يقول ال تكلمو من الخيارات انك تستشير المحامي وكذا‬
Tell the patient

✅✅
49. crown 10 and root 15?wht crown root ratio?
2:3
50. Down syndrome clinical presentation


51. pseudo class iii?
Skeletal class one
✅✅
Dental clas one molar shifted to anterior cross bite
52. class 2 what used?
Head gaer if the mandibular normal
✅✅
53. How we can use the provisional crown as diagnostic ?

✳️✳️
Diagnosis taper
Disagnosis occlusal clearance
Hypersensitivity and reversible pulpits

✅✅
54. Chose dental floss depends on:
Proximal tights
Proximal surface roughness
Ease to use and preference

55. Definition class 1 malocclusion?

✅✅
Mesiobuccul cusp of upper first molar occlude to buccul grove of lower first
molar
56. Patient come after 2 days of amalgam restoration complain about

✅✅
discoloration of this tooth the cause of discoloration ?
Extensive tooth prep
Deficient in bolishing

57.

‫عالج‬ ‫ ايش‬desqymative gingivitis‫؟‬

Despite the availability of many therapeutic agents that claim to reduce


severity, no intervention that is completely successful for treatment of
desquamative gingivitisexists. The gingival lesions are usuallytreated by

✅✅
improved oral hygiene measures and occlusive topical and systemic
corticosteroid therapy
14 years old patient
Parents complain his
12 eruupted buccaly
13 missing
23 errupted 9 months ago
So what type of bridge needed ?

✅✅
A-No treatment
B-Resin bridge
C-Conventional bridge

✳️✳️59. The worst tooth prognosis in periodentits case :


A. Upper first molar ✳️✳️➡️ more accumilation plaque

B. Lower first molar


C. Upper first premolar
D. ‫مش فاكر الخيارات‬

Dr.Afnan
‫اذكروني بدعوة‬
‫ مارس‬٤ ‫مناقشة‬

✅✅
1. Picture of sickle scalers ‫ وفي ايش تستخدم الجواب‬calculus supra-gingival
‫‪ .2‬اختبار اليد‬
‫️✳️✳‬
‫‪ .2‬حالة للينجوال بالت انو في ميالن وحركة بسيطة باالسنان واللي باقي االنسايسر‬
3. flat brown pigmentation in buccal mucosa in african women lap result epithilial

✅✅
bulla what is the diagnosis?
oral melanotic macule
✅✅
4. girl forced to do orthodontic treatment what called in ethic dentistry?
Against autonomy

** Autonomy is the patients right to know his treatment and right to decide.

5. why patient do orthodontic treatment?


can help prevent or improve periodontal problems, can help prevent and reduce
further bone loss around teeth, improve the dentist's chances to restore missing

✅✅✅
teeth, adjust aesthetics to get a better smile and facial appearance, enhance
function of teeth, increase self-confidence
6. when deciding to do RCT over PFM which bur is used to cut porcelain and

➡️ ✅✅
which cut metal?

➡️ ✅✅
Porcelien diamond bur
Metal carbide

✅✅
7. PT spit in the chair type of disnfection?
Follow manufacturer instructions

🌸 amalgam ( any toxic material )— considering as Hazard waste


🌸extracted tooth — should put it in medical/ dental waste (incinerated)
* unless theres amalgam — put it in fixure or special container ( non incinerated )

🌸infectious waste — blood , and sharp instrument

🌟
————————————————————

🌟
pt vomit on the chair > follow instructions
pt vomit on the floor >intermediate
extracted tooth and pt. want to take it : nothing

👻vomit + blood : infectious


🍃Vomiting on the floor: intermediate level disinfection
🍃Towel used to clean vomiting: biohazards waste
🍃Vomiting is considered contaminated
🍃dental chair we use intermediate level disinfection in all situations.
🍃TB pt under treatment: we can work for him after 2wks from starting ttt
✅✅✅✅
8. sharp instrument after surgery where to put?

17 year - old man is undergoing surgical extraction of his molar tooth , the
procedure went with no complications . Which of the following ways the sharps
used in this case should be managed?
A. cleaned for future use
B. Placed in the recycle pin

✅✅
C. sterilized for the next patient
D. Disposed in puncture resistant container

🌟
9. extracted tooth where to put?
extracted teeth (infectious)
-For training:
amalgam>formalin
No amalgam>autoclave
-For disposal:
Amalagam > container labeled “recycle”
( biohazard waste )
No amalgam > container labeled “medical waste”

✅✅✅
towel : hazard

✅✅
10. class ii div 2 treatment in children with crowding arch?
Extraction of upper 4s

11. other question about the order of treatment of child have crowding and class
2
I didn't rember the exact question they want the first thing to do ?

**8 y Girl patient came wit her parents to clinic , she have crowding and not
erupted canine and she
have deficiency of maxilla and prognathic mandible? What you will see first ??
A. crowding

✅✅
B. not erupted canine
C. maxilla deficiency
D. prognathic mandible

☝🏼☝🏼
‫سؤال يشبهو‬

✅✅
12. percentage of recurrence of pyogenic granuloma ?
15%
✅✅
13. effective anaethesia in childern?
lidocain 2%
14. patient have cancer he is on chemo therapy and a symtomatic decayed tooth
what to do and the lap result was not good ?


‫بسأل من عالج السن ؟‬
Dont extraction

✅✅
15. enchorage good behavior of child ?
Positive reinforcement

+ve .. ‫تشجيع ايجابي بالكالم او هديه‬


-ve .. ‫ممكن رفع نبرة الصوت فيقوم يهدا الطفل‬

16. 8x5 well-defined radiolucency in body of mandible painless associated with


vital teeth, what is the management?

➡️ ✅
➡️ ✅✅
‫ل‬if children marsipulization
Old pt. Surgical resection

➡️ ✅✅
17.best radigraph to locate impacted canine?

✅✅
The best CBCT
then occlusal
18. 8 year old pt fully erupted right central and missing left central with no history
of truma what is the cause of missing?

May be congenital or mesidense ✳️✳️


👍🏾‫ حتوضح اكتر‬xray ‫اذن ع حسب تفاصيل السؤال اذا في او ال واذا في‬
19. pic of lefort 3 fracture?

20. pic of notch incisor what is the cause?


Congenital syphilis ✅✅

**‫لو سأل على الميكروب‬


lesion hypoplasia
Or amelogenesis

✅✅
21. treatment of squamous cell carcinoma?
Radiotherapy
Treatment. For most oral cavity cancers, surgery is the initial treatment of choice.

✅✅
Radiation or chemoradiation is added postoperatively if disease is more
advanced or has high-risk features

**Resection
Radiotherapy
Chemotherapy

22. Question about new perio classification.

➡️‫ فروع‬IANB ??
‫‏‬insisive + mental ✅✅
➡️‫ موالر منطمر مع كيس حوله والتشخيص‬3 ‫اجت حالة لمريضة تعاني من ألم في جهة وحاطين صورة فيها‬
‫ لأللم مكتوب في السؤال انو‬Irreversible pulptes ‫لسن تاني مكتوب اول شي بعملو‬..
‫الخيارات‬
1‫انو اعمل معالجة اندو فقط‬..

✅✅
2‫وال اخبر المريض بوجود موالر منطمر‬..
‫مع االفة‬

patient came with irreversible pulpitis , ( give me panorama with large lesion in 37
and 38 region , and have impacted third ) :

✅✅
Treat endo only
Tell patient about the lesion
Discusses to extract impacted third molar

➡️ ‫🏾👇🏾👇صورة اشعه قريبه لدول‬

✅✅✅
➡️ question about clefts?
➡️ question about chlorhexidine chip?
Question about bone defect?
➡️ question about stafnes cyst?
➡️ question about space infection?
➡️ question about band pusher?

Band pusher .. initially seat the band by dentist ✅✅


Dr.Afnan
‫اذكروني بدعوة‬
‫ مارس‬٦ ‫مناقشة‬

➡️➡️ Lesions of mucous membrane pemphigoid seen generally on ...


B.buccal mucosa✅✅
A.gingiva

C labial mucosa
D.palate
➡️➡️ White striae appearing on buccal mucosa ...it's
✅✅
.lichen planus
➡️➡️ A dentist got a prick injury from the patient who went hepatitis b treatment
3 months back n now normal as the pt said ..what .is the first procedure to be
followed by the dentist …

➡️➡️A .apply antiseptic to the injury


✅✅
B.apply scrub n wash n let it bleed freely
C.dentist to consult for infection diseases specialist
D.send the patient for general doctor for confirmation

➡️➡️Pt aged 36 yrs under warfarin therapy reports normally healthy but with INR
3.5 ..what's the best maximum value with in which to get the extraction on the
same appointment ?

✅✅
A.2.0
B. 3.0
C.3.5
D.4.0

✳️✳️Pt aged 24 yrs in routine checkup identified with unfinished (image x-ray)
gp filling in canine n 0 .5 cm apical radiolucency .....best option to follow up?
A. Retreat the tooth
B. Advise extraction
C.since it is not complained leave until symptoms appear
D.advise antibiotics

➡️➡️ Pt age 46 yrs female Hb1AC ..8.4 m/mol ...normal BP...with generalised
BOP...n generalized pockets of 5 to 7mm ....if advised for periodontal therapy?
Wt wd be the prognosis ?
A. Fair

✅✅
B. Good
C. Poor
➡️➡️The metal component put over the implant in the second stage implant
Healing abutment✅✅
surgery ....recalled for impression ....name of the component ?
➡️➡️Percentage of fluoride available in NaF
A. 1.23

✅✅
B. 2.26
C. 5
➡️➡️Dentigerous cyst with large radiolucency extending from almost premolar to
mid ramus ....including half erupted crown .... Treatment option wd be
A. Resection .

✅✅
B .enucleation
C marsupialization

➡️➡️Mother of a child patient after diagnosis of some carious deciduous teeth


she is interested in composite fillings but the doctor forced her n convinced her

✅✅
for dental amalgams....principle violated by the doctor ?
A. Autonomy
B. Justice
C.non malificience
D. Benificience

✳️✳️Pt with severe bruxism n generalised attrition came for restorations on


lower molars...which one wd be the best treatment to render
A. Gold resto
B. Amalgam

✳️
C. Ceramic inlay
D. Metal crown

➡️➡️Child patient age 9 yrs female ...withandibular prognathism n maxillary


retrusion ...
A. Head gear
B .chin up✅✅
C. Fixed braces
D. Twin block

➡️➡️Pt with normal health condition no percussion n pocket ....Radiolucency


extending over two maxillary premolars of size 1.5 cm ..
A. Apical cyst

✅✅
B. Periapical granuloma
C. Normal anatomy

✳️✳️Pt with already existing RPD from 13 to 23 ... With clasp on 13 .... Pt came
for 13 getting fractured ...n advsd for extraction ....best next step to follow
A.plan an implant n extend the clasp
B. Extract other canine also n give for
C. Repeat the prosthesis extending the clasp to next toith
➡️➡️Pt with history of tuberculosis infection ...now under no medical treatment
A .face mask ✅✅ ➡️(respiratory mask N95)
came for checkup ...which one is a must to wear by the dentist .

B. Face shield
C. Gloves

➡️➡️Child patient 8 years weight 30 kg .....dosage of ibuprofen ...?


30*0.25=min

✅✅
30*.5=max
7.5-15 ml

‫**نضرب الوزن مرة في نص ومره في ربع‬

➡️➡️Pic with x-ray of lateral incisor RC treated healthy ...with moderate boneless
2 to 3mm at the cervical region ..
Best way to restore ?

✅✅
A. Composite only n crown
B. Fibre post n composite core
C. Metal post n crown

➡️➡️Molar tooth with old amalgam restoration to be replaced with ceramic inlay
....main feature to be altered ...

✅✅
A. Divergence of the walls
B. convergence of the walls
C. Gingival seat extension

Path of insertion for inlay..


**Amalgam- convergent occlusally
Inlay - divergent occlusally
diverge‫ونخليه‬converge‫نغير ال‬

➡️➡️Angle of the blade to tooth to remove the calculus


B. 45 to 90✅✅
A. Less. Than 45

C. Greater than 90 but less than 105


D. Zero
➡️➡️Which maxillary teeth cause more likely the palatal abscess?
A. Canine

✅✅
B. Premolar first
C. Lateral incisor
D. Central incisor

➡️➡️Pt with HbsAg n anti HbcIgM ...


A. Acute hbv✅✅
B. Chronic hbv
C. Recovery state

**Hepatitis surface antigen means positive infected


Igm acute infection
Test use it for HBV:
􏰁HBsAg (Hepatitis B surface antigen) A "positive" or "reactive" HBsAg test result
means that the person is infected with hepatitis B. This test can detect the actual
presence of the hepatitis B virus (called the “surface antigen”) in your blood.

􏰁HBcAg (core antigen) is a hepatitis B viral protein, It is an indicator of active


viral replication; this means the person infected with Hepatitis B can likely
transmit the virus on to another person (i.e. the person is infectious) and HBcAg
is considered "particulate" and it does not circulate in the blood
􏰁HBeAg is the “soluble” or extractable part of the “core” antigen of the hepatitis B
virus (HBV). The presence of HBeAg indicates that the virus is actively
multiplying and is highly contagious.

➡️➡️Picture of patient with spaces n upper anteriors issuing laterals 12 n 22


...best treatment to render
A. Ceramic veneers

✅✅
B. Ceramic crowns
C. Ortho rx n then implant
D. Make canines as lateral in shape n close the spaces

Dr.Afnan
‫اذكروني بدعوة‬
‫ ﻣﺎرس‬٧ ‫ﻣﻨﺎﻗﺸﺔ‬

✅✅
▶bone loss around successful implant annually?
0.5
1
2

**The criteria for successful implants allow aloss of 0.2 mm per year; only in the
firstyear after implantation, a loss of 1.0–1.5 mm is allowed. Four methods were
devised to calculate the annual bone loss according to these criteria
‫ ﺑﻌﺪﻫﺎ اﻗﻞ‬١.٥ - ١ ‫ﻓﺎول ﺳﻨﻪ‬
٠.٥ ‫ﯾﻌﻨﻲ اﻗﺮب‬

▶patient close his lips on suction tip why?


Due to big suction

pressure is higher in his mouth than the suction tip ✅✅


or lower

▶Most cases liable to failure of the following designs??

Cantilever bridge restoring missing second premolar by only fist molar

Cantilever bridge restoring missing first molar by first and second molar✅✅

▶pt with aged appearance and wrinkles at angle of mouth what we should do?

✅✅
Close feeway space
open feeway space
▶clinical case about bad crown with bony proximal defect what is the most
difficult to correct:
open margin
perforation of root

✅✅
occlusal interference
bony defect

✅✅
▶Purposes of crowning RPD abutment:
to change morphology or contour
to protect form caries
to protect from fracture

✳✳▶maximum coverage distalization of denture means of what?


Support
retention
stability

**Retention .. prevent prosthesis to move away from tissue

Stability .. prevent to move in horizontal plane

Support .. prevent to move toward tissue

✅✅
▶least wear in both upper and lower restoration:
gold against gold
gold against acrylic
glazed porcelain against acrylic
glazed porcelain against glazed porcelain

▶If I have horizontal movement of the tooth 1mm according to classification of


mobility what is the class

✅✅
Class I (less than 1mm)
Class II (1-2mm)
Class II (more than 2mm)

✅✅
▶bur used for preparation of cingulum rest ?
Inverted cone
round
tapered

**A flat end, large diamond cylinder is usually


used. A safe-sided 1/4-inch diamond disc can
be used in cases, where the lateral incisor and
the premolar are absent and sufficient space
is available.
▶Dentist after Jaw relation uses a pupillometer silicon index to record something.
what is the use of pupillometer?

✅✅
Helps aligning anterior teeth to interpapillary line.

Aligning teeth to facial midline.

Helps align anterior teeth in the same position where the natural teeth existed.

Helps in arranging the teeth to camper’s line


**one used to measure the pupillary light reflex, and the other used in
ophthalmology, which measures the distance between pupils through visual
stimuli

▶according to recent research the success of the fixed denture duration

✅✅
25 years
12 to 15 years
not related to the bridge span

✅✅
▶Extension of 3/4 anterior partial veneer in proximal surface:
should be hidden lingually
should be extended labial for better cleaning

▶patient suffering from gastric acidity and..... The question was which material or
alloy is the most resistant to acidic solution
Stainless steel
Cobalt chromium

✅✅
Zirconium dioxide
Titanium palladium aluminum
▶Preparation of gold crown with expects of gingival recession, most proper extent
of preparation:
1mm under the gingival margin
Make it on the fifth
Make it on the third✳✳
At the gingival margin✳✳

▶Which of the following is true about the super-eruption of unopposed molars?


Super-eruption occurs 60% of the time.
Super-eruption is more prevalent in the mandibular arch.

✅✅
Unopposed molars have a mean super-eruption of 3.0mm.
Attachment loss is one of the main predictors
▶female patient with ridge lab designed bridge came after 1-week suffering from
pain underneath the bridge and bleeding when brushing why?
pontic pressure or closed ✅✅
gingival embrasures

▶If you implant restoring lateral anterior with short occlusal gingival Height
implant which of the following crowns should be used??

✅✅
Cemented
Screwed
Ucla

✅✅
▶most common cause of implant Screw losing one week after finishing work??
Bond resin between implant and abatement
Gingival tissue overgrowth
Using provisional restoration
Using definite permeant restoration

▶Mental foramen distance of distal side of implant anterior to it:


2

✅✅
3
5
‫‪Dr.Afnan‬‬
‫اذﻛﺮوﻧﻲ ﺑﺪﻋﻮة‬
‫ ﻣﺎرس‬٨ ‫ﻣﻨﺎﻗﺸﺔ‬

▶depth of pin in amalgam


A. 0.5

✅✅
B. 1
C. 2
D. 3


▶How to do impression for patient with gag reflex?
increase heat of Water

**Depending on type of impression


Increase the setting time
Either by adding catalyst
Of adding heat dependingon

▶Too Much Questions about Trigeminal Neuralgia


▶Pedo patient with good oral hygiene & No Caries. What's the dose of Topical

✅✅
flouride prescribed
1. No Need for Fluoride

✳✳▶pain after extraction and no infection in the Socket


- osteomylits (I choosed This but not sure)
** dry socket after 2-3 days
More than 3 days➡➡ osteomylits
✅✅
▶which Rest cause senstivity in Tooth
A. Amalgam
B. 3/4 Crown
C. Veneer
D. Veneer

✳✳▶During extraction which alveolar bone is most susceptible to fracture?


Mand incisor
upper canine ✳✳
mand post

✅✅
▶The Most material sedative The Pulp:
1 - ZoE
2 - CaoH
3 - Zn. P
4 Can't Remember

✳✳▶Many cases with Pictures and asking for the Swellings associated with the
Muscle associated

**Decayed tooth with infection, infection spread to facial spaces, what is the thing

✅✅
that determines where the infection will spread?
A-muscle attachment
B-tooth position
C- Inclination of tooth
D-type of tooth

👆🏾👆🏾
‫ﺳﺆال ﺷﺒﯿﻪ‬

**Position of muscle attachment in relation to root tip.

1-thickness of bone adjacent to the offending tooth


2-position of muscle attachment....
3-virulence of the organism
4-status of the patient immune system

‫اﻻﺷﯿﺎء اﻟﻠﻲ ﺗﺄﺛﺮ‬ 👆🏾👆🏾

▶‫ﻣﺮﯾﺾ ﻋﻨﺪه ﺣﺴﺎﺳﯿﻪ ﻣﻦ ال‬sulfer

✅✅
‫اﻟﻤﻔﺮوض اﺗﺠﻨﺐ اﺳﺘﺨﺪام اﯾﻪ؟‬
‫ﺣﻠﯿﺘﻬﺎ‬poly sulfied

✅✅
▶which class is totally tooth support?
Class 3 and 4

▶‫ﺑﺴﺘﺨﺪم اﯾﻪ ﻟﻘﯿﺎس‬


width of gingiva?
Ging width peridontal prope ✅✅

➡١١،١٢prob‫ﺑﺘﺴﺘﺨﺪم ﻻﯾﻪ؟‬

Garacy for mesial posterior teeth✅✅

▶class II with large over-jet


Extraction of upper
head gear cervical

**Depending on the age


If still growing
Functional appliances or headgear depends on the case
If growth stoped
Camouflage by extraction of upper 4s
or upper 4s and lower 5s if there is crowding

▶Two cavity in incisor small and large ‫ ﻫﻌﻤﻞ‬restor‫?ﻻﯾﻪ اﻻول‬

Larg cavity ‫اول‬

✅✅✅
Small restoration ‫اول‬

**preparation‫ﻧﺒﺪا ﺑﺎﻟﻜﺒﯿﺮ‬
‫ و ﻟﻤﻦ ﻧﺠﻲ ﻧﻌﻤﻞ‬filling ‫اﺑﺪا ﺑﺎﻟﺼﻐﯿﺮه‬

▶Patient control HIV from 10 years and you dentists


Injury?
Follow up ‫وﻻ ﯾﺎﺧﺪ اﻟﻤﺼﻞ ﺣﺎﻻ‬

Test -report -anti Hiv ✅✅

▶Upper teeth aganist complete lower denter after 8 years ridge of mand?

✅✅
Knife edge
flappy
flat

▶Kennedy classification is important

✅✅
▶Nasoplatine cyst ‫?ﺑﯿﻜﻮن ﻓﯿﻦ‬
Between two central
▶‫ﻟﻮ ﻋﻤﻠﺖ‬nerve block ‫ل‬IAN ‫وال‬lingual n
‫ ﻣﻤﻜﻦ اﺧﻠﻊ اﯾﻪ؟‬molar permolar ant ‫وﻻﻛﻞ دول‬

Premolars ✅✅

▶Flouried in dentifrices in ppm?


500

✅✅
225
1000

**1000-1450 in adult
500 in child
hcl%11‫? ﺧﯿﺎرات ﺗﺎﻧﯿﻪ ﻣﺶ ﻓﺎﻛﺮاﻫﺎ‬main componant in dental bleeching ‫▶اﯾﻪ‬

: Four possible approaches are available 💡


Acid –pumice microabrasion .1
Bleaching .2
Veneers .3
Crowns .4

📍
Acid –pumice abrasion technique
.Used only for mild staining & surface enamel opacities

.HCL acid + pumice 18% 💡

Phosphoric acid technique slower but safer💡


..for 1-2 min 30-50%

..Bleaching💡
(Vital (office and home tech
📍
Office use h2o2. 25-40%

.Home use carbamide 10-15%📍

( .Non vital (walking and inside outside tech

(Walking 35%carbamide or sodium perborate(which is best 📍


.Pt come after week

Inside-outside 10%carbamide peroxide 📍

▶‫اﯾﻪ اﻟﻲ ﺑﯿﻄﻮل ال‬setting time?


Increase water
decrease tempreture

**If it alginate then B


Decrease temperature

✅✅
▶Def. Of class 1
MB cusp of up.6 occlude in the buccul grove of lower 6
✅✅
▶Starry sky apperance?
Burkitt lymphoma

📍Burkitt lymphoma.. endemic childhood malignancy in Africa.


📍more frequently in the maxilla than the mandible.
🔆‘starry sky’ pattern in Histo..
The tumours are rapidly growing..

▶‫اﻟﺪﻛﺘﻮر ﺷﻚ ﻓﻲ ﻣﺮﯾﺾ اﻧﻪ ﻋﻨﺪه‬hbv‫؟ ﻟﯿﻪ ﺷﻚ‬

✅✅
Pale skin
yellow skin and yellow eye
✅✅
▶‫ اﻓﻀﻞ اﺷﻌﻪ ﻟﻞ‬impacted canine?
Cbct

Dr.Afnan
‫اذﻛﺮوﻧﻲ ﺑﺪﻋﻮة‬
‫مارس‬٩ ‫مناقشة‬

▶️ Bohns nodule >


3week baby

**Natal teeth usually erupt in the centre of mandibular ridge as central incisors.
They have little root structure and are attached to the end of the gum by soft

✅✅
tissue. Bohn's nodules usually rupture spontaneously and disappear within a few
weeks to a few months
** difrent between bohn’s nodules and epestien pearl
▶️‫ناحيه‬
‎ ‫ جاب صوره فك مكسور من‬symphysis ‫ وكاتب انو كم‬plate ‫نحتاج والخيارات انو واخد او اثنين او‬

2 plate✅✅
‫?ثالثه‬

▶️Bony protrubane in anterior of mandiubule > genial tubercal✅✅


Genial tubercle – The genial tubercle is a spiny protuberance or prominence
(sometimes two) of bone located in the midline on the lingual aspect of the
mandible below the roots of the incisor teeth. The genial tubercle is also referred
to as the mental spine. …
‫‪ gag‬كثيره و ‪ saliva‬في سناريو انو بيشن مريض قلب وكان بياخد ادويه كثييره وانو وانتي بتعالجي كان ‪‎‬في️▶‬
‫اي دواء ممكن يعمل الحاجات دي ؟ ‪refelex‬‬
‫‪ ace inhbator or digtal‬من الخيارات‬

‫‪Digaxion‬‬ ‫✅✅‬
‫؟ ‪metal‬االداء البتستخدم لقياس ‪‎‬ال️▶‬
‫✅✅‪iwanson caliper‬‬
‫ماحدد كلينكال او الب وكاتب الخيارين ‪vibrating line‬كيف نحدد ‪‎‬ال️▶‬
‫✅✅‪Kingsley scraper in cast‬‬
‫✅✅‪T burnisher in patient‬‬

‫✅✅‬
‫كلهاا ‪ ethic‬اسئله عن تعريفات ‪‎‬‬
‫ال‬

‫‪‎‬‬
‫نورالجيا‬ ‫✅✅ سناريو عن تراجيمنل‬
▶️‎mechanism of pencillin
Cell wall✅✅
DNA ➡️ metranidzol ✅

▶️‫عن‬‎ ‫ سؤال‬drug for pregnancy


‎‫من االستبعاد‬
Flucnzol
Clathromycin ‫?تقريبا كذه‬
Drug with pregnant:
*Contraindicated*
Asprin with high dose
Ibuprofen
Diclophenac
*Indicated*
Acetaminophen is usually use
Asprin with low dose

▶️The best descrubtion of infection control:


Decrese the transimation of pathogen✅✅
Block transmition of pathogen

‫مامتذكره‬ ‫باقي الخيارات‬
▶️Dentist performing a perio surgery , full flap with blade 15, and now he need
(forget what instrument to position flap), what’s the sequence?

✅✅
A- Nurse take the blade carefully then handle the dentist the other instrument
B- Dentist place the blade on a neutral surface for the nurse to pick

▶️‎pic of balm grasp


▶️‫ال‬‎ ‫ صوره عن عالقه‬IAN ‫ مع السن حقت ال‬perforation of groove or superimbosed
▶️‎‫ اي‬probe ‫ ورقم تاني‬4،6 ‫مافيهو رقم‬
University of Michigan with William markings ✅✅

▶️‫سينما‬
Forign body✅✅
‎ ‫سناريو حق البيشن البياكل فشار في‬

▶️
‎‫سؤال وصوره لبيشن سوشيال بوور ممكن يكون عندها شنو‬
Desqymative gingivitis
Nug
‎ ‫خيارات‬
‫تانيه‬

Socialy poor /stress and malnutritiob / NUG ✅✅


▶️‫بياخد‬
‎ ‫ بيشن‬corticoid ‫ممكن يحصل شنو ؟‬
✅✅
adrinal insuficany

▶️‎pedo patient you want to anesthetize D and E ?


IAN block
Long buccal

➡️ IAN block ✅✅
**deppinding on age if more than 7 years

▶️ scenario about space maintainer


Reverse Band and loop✅✅
‫ موجودين‬6‫تقريبا ال‬
** distal shoe if it’s still erupting to help in it’s path of eruption
‫‪band and loop‬‬ ‫** لما نخلع ال ‪ D‬ويكون ال ‪ E‬موجود️➡️➡‬
‫‪ = band and loop‬موجود ‪ E‬وكان ال ‪ D‬إذا خلعنا‬

‫)هذا في حالة ال‪ 6‬طالع(‪ = reverse band and loop‬موجود ‪ D‬وكان ال ‪ E‬اذا خلعنا‬

‫‪ distal shoo‬يبقى نسوي ‪E.‬اما لوكان ال‪6‬ما طلع وخلعنا‬

‫و االسنان الدايمه ال ادام طلعت ‪e bilateral‬كمل بقا لو خلعت‬


‫‪Lingual arch‬‬

‫‪ ..‬ولو خلعنا ‪ uper ant‬بنستخدم ‪👍🏻nance apliance‬‬


‫✅✅✅✅✅✅✅✅✅‬
▶️‫ال‬‎ ‫ انواع الكالسب بستخدمه مع‬midbucula undercut‎‫اليستخدم مع شنو؟‬
RPI✅✅

Aesthatic + tilited + more stress + misolingual undercut = ring

Esthatic + mid buccal = Rpi


Undercut I bar
Mesiofacial undercut = worit wire
Rotation root = revers clasp
Class l + good oral hygiene = aker

▶️why do we ask patient to rinse before starting treatment?


to remove any reminant of food✳️✳️
Reduce microorganism in mouth✳️✳️

▶️Class1 >mallaigment✅✅

▶️Why use ortho >ethatic✅✅


▶️Use the glazed with >
compiste

✅✅
Compemer
Gic to prevent dehydration
‫‪Dr.Afnan‬‬
‫اذكروني بدعوة‬
‫ مارس‬١٠ ‫مناقشة‬

✅✅
1.pic of nabers probe and its use ?
Used for forcation involvement

✅✅
2.HIV pt with linear erythematous inflamation on marginal gingiva .. how to
manage ? Fluconazole i got this q twic

3.autism patient from whom you will take consent form ?

✅✅
Patient
father
brother

✅✅
4. dentist want to encourage pedo pt for his good behaviour ?
Positive reinforcment

5. management of ANUG?
✅✅
If with pseudomembrane ..remove it with sterile cotton rolls then
subragingival scaling

‫ اول ستيب اساسا‬remove membrane then scaling supragingival then prescribe

➡️ ✅✅
mouth wash and Antibiotic+ metronidazole to prevent to reccurance
After healing sup gingival

6. Two decayed teeth distal of first and mesial of second premolars and
need composite . Why its prefered to do it same visit ?

✅✅
Better marginal adaptation for both
‎ ‫طبعا بديهي اخترت هالجواب الباقي مالو عالقه‬
‫ابدا‬

‫هذي الصوره السؤال‬


How much is the keratinized tissue measrment?

✅✅
Place your probe on the outside of the tissue and measure from the
gingival margin to the mucogingival junction.
‫ مم‬٣ ‫الصوره باالختبار اوضح من هذي و الرقم مع ماركيس بروب طلع لي‬

✅✅
7. What is the antidot of heparin ?
Protamin sulfate
8. patient with class 2 and hyper-divergent mandible .. which appliance ?
Cervical headgear

✅✅
Straight headgear
High pull
Low pull

✅✅
10. pic of band pusher and its use?
Initially by dentist
✅✅
11.location of mandibular foramen in adult ?

➡️➡️
Above
below occlusal plane in child

12. pic xray of internal resorption shape


✅✅
13. bleach in endo treated tooth lead to ?
Cervical external resorption
Internal resorption
Replacement resorption

➡️
➡️
**Vital tooth Apical periodontitis
Non vital tooth cervical resorption

✅✅
14. uses of sickle scaler ?
Supragingival scaling
15. pressure in autoclave when its 121 C temperature ?
10
15
20
5
✅✅
**125 =16 minute
121=24 minute

16. mother bring her son 8 yrs with avulsed upper central before 15 mins

✅✅
how to manage ?
Replant
Replant and then pulpotomy
Replant and then pulpectomy
17. 8 years had trauma before one day to his upper central ..enamel dentin
and pulp involved whats your management ?
Pulpectomy

✅✅
Pulpotomy
Apexogenisis

✅✅
18. why to rinse the mouth before start treatment ?
Decrease microflora

✳️✳️19. patient 20 yrs with generalized bleeding and localized attachment


loss around some teeth ? How to manage ?
All choices starts with scalling but differs in antibiotic or chlorhexidine
mouth wash or local minocycline

20. distal extension of lower denture (how it would be covering retromolar


pad ?
Full

✅✅
or one third
or two third
21. 14 year boy lost his upper right central due to trauma and its non
restorable now
He has good oral hygiene and has all his teeth sound .. he want to replace
central with immediate denture how many clasps you make ?
1
2

✅✅
3
None

**spoon denture

22. disease has high melanin somthin ! Makes black to brown color on
gingiva ?

✅✅
Heamangioma
Addisons dis
✅✅
23. X-ray shows strip perforation?
PA

✅✅
24. ramsay palsy and the causative organism?
Varcella zoster virus
✅✅
25. lesion in lower anterior mandible with vital teeth ?
Cementoosseous dysplasia
Fibrous dysplasia

➡️ ✅
**multifocal radiopaque with narrow rim radiolucent on lower vital anterior
teeth cemento osseous dysplasia
26. patient on pisphosphobate since years want to treat unrestorable tooth
Rct ✅✅
Extract

**Risk of necrosis with the extraction

27. patient with 10 mg prednisone want to do simple extraction ( repeated


many times)
28. Definition of deontology?
is the normative ethical theory that the morality of an action should be
based on whether that action itself is right or wrong under a series of rules,
rather than based on the consequences of the action.
It is sometimes described as duty-, obligation-, or rule-based ethics. ✅✅
✳️✳️29.x ray shows crown with open margin mesial and distal whats the
cause ?
Core expansion
Casting shrinkage
Two other choices i dont remember

30. Question about porcelin chipping in pfm crown what could happen
In manal notes you can find

**Chipping of porcelain = deformation in design and location of metal


porcelain junction

👆🏾
‫منال‬

31. pic of cast left side with distal extension the abutment tooth to be used
is second premolar and its mod amalgam restored how you will use it as
abutment ?

✅✅
Dont use it as abutmenf
Survey it with crown
Prepare rest within amalgam ‫يعني الخالصه كيف تستخدم سن محشي املغم م او دي ك ابتمنت ؟‬

✅✅
33. definition of pulpotomy?
Removal of coronal pulp
✅✅
34. cell in periodontium responsible about bone remodelling?
Fibroblast

**responsible for bone resorption & deposition 👉👉 osteoclasts


responsible for collagen remodeling after orthodontic treatment👉👉
fibroblasts
35. what is the procedure name remove soft and hard tissue

✅✅
Curetage
Root planning
Scalling

Dr.Afnan
‫اذكروني بدعوة‬
‫ ﻣﺎرس‬١٢-١١ ‫ﻣﻨﺎﻗﺸﺔ‬

‫ﻣﺎرس‬١١

1_The best characteristic in ceramic


A-opaque

✅✅
B- transparent
C-translucent
‫اﻟﺮاﺑﻊ ﻧﺴﯿﺘﻮ‬..

2-
Fluoride varnish contraindicated with:

✅✅
A-hepatic failure
B-renal failure
C-cerberal something may be palsy
D-chemotherapy and radiotherapy

3-Picture of hatchinson incisor of congenital syphilis


‫✅✅‬
‫‪4-How to disinfect wax occlusal rim?:‬‬
‫‪Something phenols‬‬
‫ا‬

‫‪5-‬‬
‫ﺳﯿﻨﺎرﯾﻮ ﻋﻠﻰ اﻻﻣﺒﻼﻧﺖ‬
‫‪ torque=35NCM‬اﻧﻪ ﺑﯿﺸﻨﺖ ﻋﻤﻞ اﻣﺒﻼﻧﺖ وﻛﺎﻧﺖ ال‬
‫ﻋﻨﺪه ‪ eccentric movement‬وﻛﻤﺎن ﺑﺎل ‪ slight contact‬ﻋﻨﺪه ‪ maximum inter cuspation‬وﻓﻲ ال‬
‫ﻛﻮﻧﺘﺎﻛﺖ‬
‫اﯾﺶ اﻟﺴﺒﺐ؟ ‪ loose‬رﺟﻊ ﺑﻌﺪ ﻓﺘﺮك وﻋﻨﺪه‬
‫ﺑﺴﺒﺐ اﻟﺘﻮرك‪ ..‬او اﻟﻜﻮﻧﺘﺎﻛﺖ ﺑﺎﻟﻤﺎﻛﺴﻤﻢ اﻧﺘﺮﻛﺴﺒﯿﺸﻦ‪ ..‬او اﻻﻛﺴﻨﺘﺮك ﻣﻮﻓﻤﻨﺖ‬
‫✳✳اﺑﺤﺜﻮ ﻛﯿﻒ اﻻﻛﻠﻮﺟﻦ واﻟﺘﻮرك ﻟﻼﻣﺒﻼﻧﺖ‬

✅✅
6-Best tear resistance material?
‫اﺧﺘﺮت‬Polysulphide

✅✅
Autoantibodies?
Systemic lupus ‫اﺧﺘﺮت‬erythematous

8-Case about a patient having a complete dentue but after time the denture not

✅✅
fit in his mouth :
Paget disease

✅✅
9-Orange peel appearance
Fibrous dysplasia
‫اﺧﺘﺮت‬

✅✅
10-Patient with muco cutaneos lesion and prickle cell:
Bullous ‫اﺧﺘﺮت‬Pemphygoid

11-

✅✅
How much the irrigation needle must go insude the canal:
2mm before apex
4mm before apex

✳✳12-
Which sterlization don't cause corrosion of burs:

✅✅
Autoclave
Dryoven
Ethilene oxide ir something like that
Chemical vapor sterlization
Many Many questions about implant

U have a distance of 14mm how many implants u will use ➡ 2 ✅

Uhave a distance i think 10 mm and ther is a picture including different implant


diameter and u have 2 choose one of them?

**10 ‫ﻋﻨﺪك ﻣﺴﺎﻓﺔ ﺑﯿﻦ اﺛﻨﯿﻦ اﺳﻨﺎن‬mm


‫وﻋﻄﺎﻛﻲ اﻣﺒﻼﻧﺖ ﺑﻤﺨﺘﻠﻒ اﻟﺪﯾﺎﻣﯿﺘﺮ‬

✅✅
?‫اﺧﺘﺎري اي ﻣﻨﻬﻢ ﺑﺘﺨﻄﯿﻬﺎ‬
mm 6,5 ‫اﻗﺮب اﺟﺎﺑﺔ‬

✅✅
Implant stability rfa➡ RFA is a measure for implant stability

Implant and bone ➡➡osseointgration ✅✅


Picture and ask what the name of this part of implant!

Cover screw

✅✅
Fixture
Implant abutment
Impression coping

‫ﺟﺎب ﺻﻮرة ﻛﺪا ﻓﻲ ﺣﺒﻮب ﺻﻐﯿﺮة ﺗﺤﺖ اﻟﺸﻔﺎﯾﻒ ﺑﺎﻟﻜﻮرﻧﺮ‬


Herpangina
Herpes zoster

✅✅
Primary herpes..
Reccurunt herpes
✳✳Senario of patient u made for him a deep mesio oclusso distal preparation
and amalgam restoration than he came back with complete loss of sensation and
numbness in the area-upper right- what's the cause:
Anesthesia into the sinus
Broken needle
Allergy from anesthesia
I forgot the last one

✅✅
?‫ﺟﺎب اﻧﻪ ﻋﺎﻣﻞ ﺣﺸﻮة اﻣﻠﺠﻢ ﻛﻼس ﺗﻮ وﻛﻠﯿﻨﻜﻞ ﻣﺎﻓﯿﺶ ﺷﻲ رادﯾﻮﺟﺮف ﻓﻲ ﻻﯾﻦ ﻣﻤﺘﺪ ﻋﻨﺪ اﻻﺳﺘﻤﺲ اﯾﺶ اﻟﺘﺸﺨﯿﺺ‬
Crarked tooth

Questions about Hba1and sometimes gives you the range:


‫وﯾﻘﻮﻟﻚ ﺗﺸﺘﻐﻠﻪ وﻻ ﺗﺤﻮﻟﻪ‬
‫اﺟﺖ ﯾﻤﻜﻦ ﺛﻼث اﺳﺌﻠﺔ ﻣﺶ ﻣﺘﺬﻛﺮة اﻟﺴﯿﻨﺎرﯾﻮ ﺑﺲ اﻟﺨﻼﺻﺔ ﻣﺘﻰ ﺗﻘﺪر ﺗﺸﺘﻐﻠﻪ وﻣﺘﻰ ﻻ‬
‫ﯾﻌﻨﻲ اﻋﺮﻓﻮا ﻛﯿﻒ ﻧﺘﻌﺎﻣﻞ ﻣﻌﻬﻢ‬
‫ﺟﺎب ﺳﺆال ﻛﯿﻒ ﺗﻘﯿﺲ اﻟﻤﻮﺑﻠﺘﻲ اﻟﻨﺎﺗﺠﺔ ﻣﻦ اﻻﻛﻠﻮﺟﻞ ﺗﺮاوﻣﺎ؟‬
‫‪Mobility test‬‬
✅✅Fermitus test
‫ﺟﺎب ع اﻟﻨﯿﻮ ﺑﺮدوﻧﺘﻞ ﻛﻼﺳﻔﻜﯿﺸﻦ ﺳﻮال ﻓﻘﻂ‬

‫ ﻗﺒﻞ ﺳﺖ ﺳﻨﻮات وﻋﻨﺪه ﺟﻨﺠﻔﺎل‬kidney transplantatiin ‫ﺳﯿﻨﺎرﯾﻮ ﻃﻮﯾﻞ ع ﻣﺮﯾﺾ ﻋﻨﺪه ﺿﻌﻂ ووو وﻋﻤﻞ‬
‫اﻧﻼرﺟﻤﻨﺖ اﯾﺶ اﻟﺴﺒﺐ‬
Amelodipine

✅✅
Nifedipine
Cyclosporin
Phenytoin

white lesion can rapped off ‫ وﻋﻨﺪه‬systemic lupus erythematous ‫ﺟﺎب ﺳﯿﻨﺎرﯾﻮ ﻣﺮﯾﺾ ﻋﻨﺪه‬
‫ ﻛﯿﻒ ﺗﻌﺎﻟﺠﻬﺎ‬and erythema base

✅✅
‫ﻓﻲ ﻛﺎن ﻛﻮرﺗﯿﺰون ﺗﻮﺑﯿﻜﺎل‬

🏼
‫وﻧﺴﺘﺎﺗﯿﻦ ﻣﺎوت ووش‬
Nystatin is antifungal ☝
‫واﻟﺒﺎﻗﻲ ﻧﺴﯿﺖ‬

✅ ‫اﺧﺘﺮت ﻧﺴﺘﺎﺗﯿﻦ ﻣﺎوت ووش ﺑﺎﻋﺘﺒﺎر اﻧﻪ ﯾﺎﺧﺪ ادوﯾﺔ ﻣﺜﺒﻄﺔ ﻟﻠﻤﻨﺎﻋﺔ ﻻﻧﻪ اوﺗﻮ اﻣﯿﻮن دﯾﺰﯾﺰ ﻓﺘﻌﻤﻞ ﻛﺎﻧﺪﯾﺪا‬

pace ‫ ﻟﻤﺮﯾﺾ ﻋﺎﻣﻞ‬ept ‫اﺟﺎ اﻟﺴﻮال اﻟﻲ ﺑﺬﻫﺒﻲ اﯾﺶ اﻟﻔﯿﻠﺮ اﻧﻪ ﻣﺤﺪدﻧﺎش ورﻛﻨﺞ ﻟﻨﺚ ﻣﻀﺒﻮط واﺳﺘﺨﺪﻣﻨﺎ‬
maker
‫ ﺟﺎب اﺷﻌﺔ ﻣﻊ اﻟﺴﺆال‬..‫اﻧﻲ اﺧﺘﺮت اﻧﻪ اﻟﺮروت ﻛﻮﻧﻔﻜﺮﺟﺸﻦ ﻻﻧﻪ ﻣﺮة ﻛﯿﺮف‬
...‫اﻟﻤﻄﻠﻮب ﺑﺎﻟﺴﻮال اﻧﻪ ﻟﯿﺶ ﻣﺎﻗﺪرﻧﺎ ﻧﺤﺪد اﻟﻮرﻛﻨﺞ ﻟﻨﺚ ﻣﻀﺒﻮط‬

‫ ﻣﻊ اﻟﺒﯿﺲ ﻣﯿﻜﺮ ﺑﺲ دا ﻣﺶ ﺳﻮاﻟﻪ‬ept ‫ﻫﻮ ﺻﺢ اﻧﻪ ﻣﻤﻨﻮع‬

A 70-year-old man presented for root canal treatment of tooth #47 (see .19
periapical radiograph). Upon examination, #47 is sensitive to percussion. Patient
had pacemaker several years ago. Working length was inconclusive with EAL.
?Which of the following is the cause of difficulties in working length determination
A. Difficult root anatomy
B. Presence of apical periodontitis
C. Presence of Full Coverage Metallic Restoration
✅✅D. EAL should not be used with patient had such madical history
EAL▶Electronic apex locater **

✅✅
RFA used for➡
Chek implant stability
Or
Bone loss around implant

➡pic of space infection i think it was pterygomandibular space


‫✳✳ع ﺣﺴﺐ اﻟﺼﻮرة‬

➡patient want to take his extracted tooth .. give and do nothin✅✅

✅✅
➡swelling on area of paritid gland 4*5 what to do?
Fine needle aspiration
Marsupialization
Incisional biopsy

✳✳➡why polymethyle metahcrylate wear with time ?


Low modulus of elasticity
High stiffness to fracture
‫ﻻ اذﻛﺮ ﺑﺎﻟﺘﺤﺪﯾﺪ‬

➡micro organism in apical periodontitis ?

✅✅
Virus
Bacteria
✅✅
➡patient spit on dental chair , disinfect how ?
Follow instruction

✅✅
➡-Crown Calcification of Permanent Teeth Complete after Eruption by :
A-6 months
B-12
C-24
D

‫ ﻣﺎرس‬١٢ ‫ﻣﻨﺎﻗﺸﺔ‬

✳✳‫ ﻣﺮﯾﺾ ﺧﻠﻊ ﻗﺒﻞ ﻓﺘﺮة ﯾﻮﻣﯿﻦ او ارﺑﻊ ﻣﺶ ﻣﺘﺬﻛﺮة وﺟﺎء ﻋﻨﺪه‬bleeding ‫ وﻛﺎن‬oozing ‫وﻋﻨﺪه‬
petechiae ‫ﻓﺎﯾﺶ ﻣﻦ ﻓﺤﺺ اوﻟﻲ ﺗﻌﻤﻠﯿﻪ‬
Inr
Rbc
Ptt
Pt

?‫ اﯾﺶ اﺳﻤﻪ‬platlet ‫اﺧﺘﺒﺎر ال‬


The platelet function analyzer (PFA)-100 is the most widely used global test
of primary hemostasis. It performs an in vitro test of platelet plug
formation, referred to as the PFA, by measuring the time to occlusion of a

✅✅
window in a coated membrane through which blood is forced at high shear
rate.

✅✅
What is the bacteria mostly found in abscess!
Anaerobic
Aerobic
Mixed
‫ﺳﯿﻨﺎرﯾﻮ ع ﻣﺮﯾﺾ ﻋﻨﺪه ﺟﯿﻨﺠﻔﺎﯾﺘﺲ ﺑﺎﻻﻧﺘﺮدﻧﺘﺎل واﻟﻤﺎرﺟﻨﺎل‬
✅✅
‫اﯾﺶ ﺗﻘﻮل ﻋﻠﯿﻪ؟‬
Marginal gingivitis
Diffused gingivitis

✅✅
!‫ﺟﺎب ﺳﺴﻨﺎرﯾﻮ ع ﺑﺮاﯾﻤﺮ اﻧﺪو ﺳﻜﻨﺪري ﺑﺮﯾﻮ ﻛﺪا ﯾﻌﻨﻲ اﻓﻬﻤﻮا ﻋﻨﻬﻢ‬
Primary endo secondary perio

✅✅
Erythema in which stage
Early stage

Which of the following is a side effect of excessive listrene in mouthwash


use?
A . lichen planus
B . morsicartio buccarum

✅✅✅
C . erythrema multiforme
D.epithelial desquamation

Which sealer least stable?


Bioceramic
‫✅✅‬
‫‪Caoh‬‬
‫‪Zoe‬‬

‫✅✅‬
‫?‪Weakest phase in amalgam‬‬
‫‪gamma-2‬‬

‫‪Patient with complete denture his free way space is 1mm‬‬


‫ﻣﻦ ﺿﻤﻦ اﻟﺨﯿﺎرات ﺗﻌﯿﺪي اﻟﺪﻧﺸﺮ‬

‫✅✅‬
‫وﻻ ﺗﻌﺪﻟﻲ اﻟﺒﯿﺰ ﺗﺒﻊ اﻟﺪﻧﺘﺸﺮ‬
‫او ﺳﻠﻜﺘﻒ ﺟﺮﻧﺪﻧﺞ ﻟﻼﻛﻠﻮﺟﻦ‬

‫ﻧﺎﻗﺺ ‪**Free way space‬‬


‫ﻣﻌﻨﺎﺗﻪ اﻧﻪ اﻧﻜﺮﯾﺰ ﺑﺎل ‪vd of occlusion‬‬

‫ﺳﺆال اﺧﺮ‬
‫ﻣﺮﯾﺾ ﻋﻨﺪه ﻧﺎﻗﺺ اﻟﻔﺮي وي اﺳﺒﯿﺲ‬
‫ﻛﯿﻒ ﺗﻘﺪري ﺗﺘﺎﻛﺪي؟‬
‫واﺣﺪ ﻣﻦ اﻟﺨﯿﺎرات ﻛﺎن‬
‫ﺗﻨﻘﺼﻲ‬

‫✅✅‬
‫‪Vd rest from Vd occlussion‬‬
‫اﺧﺘﺮﺗﻪ‬
‫‪**Free way space=vd of rest-vd of occlusion‬‬

‫✅✅‬
‫اﺟﯿﺘﻲ ﺗﻌﻤﻠﻲ ﺑﺮﺑﺮﯾﺸﻦ ﻟﻠﺴﻦ وﺑﺘﺴﺘﺨﺪﻣﯿﻬﺎ ﻟﻠﻔﻜﺴﺪ وﻗﺮﺑﺘﻲ ﻣﻦ اﻟﺒﻠﺐ ﺑﺤﯿﺚ ﻃﻠﻊ ‪ pink shadow‬اﯾﺶ ﺗﻌﻤﻠﻲ?‬
‫ﻛﺎن ﻓﻲ ﺧﯿﺎر ‪intentional RCT‬‬

‫✅✅‬
‫اﺟﺎ ﻟﻤﺎ ﺗﻌﻤﻠﻲ ‪ excessive flaring‬اﯾﺶ ال ‪ mishap‬اﻟﻲ ﻣﻤﻜﻦ ﺗﺼﯿﺮ?‬
‫‪Strip perforation‬‬
‫✅✅‬
‫اﺟﺎ ﻋﻦ اﻓﻀﻞ ﻣﺰﯾﻞ ﻟﻠﺠﻮﺗﺎﺑﺮﻛﺎ?‬
‫‪Colropherm‬‬

‫ﺻﻌﻮﺑﺔ اﻟﺤﺸﻮ واﺧﺮاﺟﻪ ﺻﻌﺐ ‪ rigid‬ﻟﯿﺶ ﻗﻞ اﺳﺘﺨﺪاﻣﻪ اﺧﺘﺮت ﻻﻧﻪ ‪Silver point‬‬

‫**‪ corrosion‬اﻏﻠﺐ اﻟﺨﯿﺎرات اﻟﺜﺎﻧﯿﺔ ﻛﺎﻧﺖ ﺗﺤﻜﻲ ع‬


‫ﻛﻼس ﻓﺎﯾﻒ ﺻﻐﯿﺮ‬
‫اﯾﺶ اﻓﻀﻞ ﺣﺸﻮة‬
‫ﻧﺎﻧﻮ‬

‫✅✅‬
‫ﻫﯿﺒﺮﯾﺪ‬
‫ﻓﻠﻮوﺑﻞ‬
‫ﺟﻼس‬

‫✅✅‬
‫‪Which upper major connector has high elasticity‬‬
‫‪Horse shoe‬‬

‫ﺗﺴﺘﺨﺪم ﻻﯾﺶ؟ ‪ prophy jet‬اﺟﺎ‬


‫رﻣﻮف ﻛﻠﻜﻠﺲ‬

‫✅✅‬
‫ﺳﻤﻮت اﻣﻠﺠﻢ اﻧﺪ ﻛﻮﻣﺒﻮزﯾﺖ‬
‫ﺑﻮﻟﯿﺶ ﻟﻼﻣﺒﻼﻧﺖ‬
‫✅✅‬
‫اﺟﺎ اﻻﻣﺒﻼﻧﺖ ﺗﻌﻤﻠﯿﻠﯿﻪ ‪ cureettage‬ﺑﺎﯾﺶ اﺧﺘﺮت ﺣﺎﺟﺔ ﺑﻼﺳﺘﯿﻚ ﻛﻨﺖ ﻗﺮاﺗﻬﺎ ﺑﻤﻜﺎن اﻧﻪ ﻧﺴﺘﺨﺪﻣﻪ ﻣﻌﻬﻢ‬
‫وﻣﺎﻧﺴﺘﺨﺪم اﻻﺳﺘﯿﻞ اﻟﻌﺎدي ﻻﻧﻬﺎ ﺗﺨﺮب‬

‫✳✳اﺟﺎ ‪ c shape canal‬ﺑﺎﯾﺶ ﺗﻌﻤﻠﻲ ‪rephinig and location of canals‬‬


‫‪BUR #1‬‬
‫‪BUR #2‬‬
‫رﻗﻢ ﺑﯿﺮ ﻃﻮﯾﻞ ﻛﺪا ﺗﻘﺮﯾﺒﺎ ‪ 602L‬ﻣﺶ ﻓﺎﻛﺮة‬
‫اﻟﺘﺮاﺳﻮﻧﯿﻚ‬

‫ﻛﻨﺪي ﻛﻼﺳﻔﻜﯿﺸﻦ‬
‫ﺟﺎب ﻛﻨﺪي ﻛﻼس ون‪ ..‬وﻋﻨﺪه ﺗﻮ ﻣﻮدﻓﻜﯿﺸﻦ‪ ..‬اﻟﻤﻮﻟﺮس واﻟﺴﻜﻨﺪ ﺑﺮي ﻫﻢ اﻟﻤﻔﻘﻮدﯾﻦ‪..‬‬
‫اﻟﻔﯿﺮﺳﺖ ﺑﺮي ﻣﻮﺟﻮدﯾﻦ واﻻﻧﺘﺮﯾﻮرس ﻣﻮﺟﻮدﯾﻦ‬
‫اﯾﺶ ﺗﻌﻤﻠﻲ?‬

‫✅✅‬
‫اﺧﺘﺮت ﻧﻌﻤﻞ رﺳﺖ ع اﻟﻤﯿﺰﯾﺎل واﻟﺪﯾﺴﺘﺎل ﻣﻦ ﻛﻞ ﺟﻬﺔ‬
‫ﺑﺎﻗﻲ اﻟﺨﯿﺎرات ﻛﺎﻧﺖ ﻏﯿﺮ ﻣﻨﻄﻘﯿﺔ زي ﻧﺨﻠﻊ اﻟﻔﯿﺮﺳﺖ او ﻧﺮﻛﺐ اوﻻ ﻟﻠﻤﻮﻟﺮس واﻟﺒﺮي وﺑﻌﺪﯾﻦ اﻻﻧﺘﺮﯾﻮر‬
‫‪Dr.Afnan‬‬
‫اذﻛﺮوﻧﻲ ﺑﺪﻋﻮة‬
‫ﻣﻨﺎﻗﺸﺔ ‪ ١٣‬ﻣﺎرس‬

‫ﺧﻼل ‪ instrumentation‬اﻧﻜﺴﺮ اﻟﻔﺎﯾﻞ ﻣﻦ اﻟﺒﺪاﯾﺔ ﻓﻲ اﻻﺑﯿﻜﺎل ﺛﯿﺮد ﺣﺎوﻟﺖ ﺗﻌﻤﻞ ﺑﺎي ﺑﺎس وﺗﺨﺮﺟﻪ ﻣﺎﺧﺮج‬
‫اﯾﺶ ﺗﺴﻮي؟‬
‫اﻛﺴﺘﺮاﻛﺸﻦ‬
‫وﻻ ﺟﺮاﺣﺔ‬
‫وﻻ ﺗﺤﺸﻲ ﻓﻮﻗﻪ✳✳‬

‫✅✅‬
‫؟‪Tooth formation in the embryonic‬‬
‫‪6 weeks iu‬‬

‫ﺑﯿﺸﻨﺖ اﺟﺖ ﺑﻌﺪ اﻧﺘﻬﺎء ﻣﻮﻋﺪﻫﺎ ﺑﻨﺺ ﺳﺎﻋﺔ ﻛﻠﻤﺖ اﻟﺴﻜﺮﺗﯿﺮ اﻧﻬﺎ ﺑﺘﺪﺧﻞ وﻻ ﺑﺘﺒﻠﻎ اﻟﺠﻬﺎت اﻟﻤﺴﺆوﻟﺔ وﻛﺎن‬
‫اﺳﻠﻮﺑﻬﺎ ﻣﺮة ﻫﺠﻮﻣﻲ‪ ،‬دﺧﻞ اﻟﺴﻜﺮﺗﯿﺮ ﯾﻜﻠﻢ اﻟﺪﻛﺘﻮر رد ﻋﻠﯿﻪ اﻟﺪﻛﺘﻮر اﻧﻪ ﻣﺎﺑﯿﺸﺘﻐﻞ ﻟﻬﺎ‪ ،‬اﯾﺶ اﻟﻲ ﯾﺒﺮر ﻣﻮﻗﻒ‬
‫اﻟﺪﻛﺘﻮر؟‬

‫اﻧﻬﺎ ﺗﺎﺧﺮت ع ﻣﻮﻋﺪﻫﺎ اﻟﻤﺤﺪد‬‫✅✅‬


‫او ﺑﺴﺒﺐ ﺳﻠﻮﻛﻬﺎ اﻟﻬﺠﻮﻣﻲ‬
‫او ﺑﺴﺒﺐ اﻧﻬﺎ اﻧﺜﻰ‬

‫✅✅‬
‫ﻟﺠﺪوﻟﻚ ﻓﺎﯾﺶ ﺗﺴﻮي‪..‬؟ ‪ vip. Patient‬اﻟﻤﺴﺆول ﻋﻠﯿﻚ اﻟﺴﻮﺑﺮﻓﯿﺠﻦ اﺿﺎف‬

‫✅✅‬
‫ﻣﺶ ﻓﺎﻛﺮة اﻟﺨﯿﺎرات ﺑﺎﻟﻀﺒﻂ ﺑﺲ اذﻛﺮ اﻧﻨﻲ اﺧﺘﺮت اﻟﺨﯿﺎر اﻟﻲ ﺗﻌﺎﻣﻠﻪ زﯾﻪ زي اي ﺑﯿﺸﻨﺖ‬
‫‪Justice‬‬

‫✅✅‬
‫‪..‬واﻋﺮاض اﻟﺮﺷﺢ وﻟﻜﻦ ﻣﺬﻛﺮش ﻓﻲ اﻟﺴﺆال اﻧﻪ ﻛﻮروﻧﺎ اﯾﺶ ﺗﺴﻮي ‪ flu‬وﺳﯿﻨﺎرﯾﻮ ﻣﺮﯾﺾ ﻋﻨﺪه‬
‫اﺧﺘﺮت ﺗﺎﺟﻞ ﻋﻼج اي ﺷﻲ ﻣﺶ اﻣﺮﺟﻨﺴﻲ‬

‫✅✅‬
‫ﺳﻮال اﻧﻪ اﺟﺎ ﻣﺮﯾﺾ ﻛﻮروﻧﺎ وﻋﺎﻟﺠﺘﻪ وﻋﻤﻠﺖ ﻛﻞ اﺣﺘﯿﺎﻃﺎﺗﻚ اﻟﻼزﻣﺔ ﻣﻌﻪ اﺛﻨﺎء ﺷﻐﻠﻚ وﻟﻜﻦ ﺑﺴﺒﺐ اﻧﺸﻐﺎﻟﻚ ﻣﺎﺑﻠﻐﺖ‬
‫ﺧﺮﻗﺖ ﺣﻖ اﯾﺶ؟ اﺧﺘﺮت اﻟﻤﺠﺘﻤﻊ‬

‫ﺑﯿﺪو اﺟﺎ وﻋﻨﺪه ‪ multiple caries‬اﯾﺶ ﺗﻌﻤﻠﻲ ﻣﻦ اﺷﻌﺔ ف اﻟﺒﺪاﯾﺔ ﺣﺪد اﻧﻪ ﻛﻔﯿﺮﺳﺖ‪..‬‬
‫اﻟﺨﯿﺎرات ﻛﺎﻧﺖ‬
‫اﻻﺷﻌﺔ اﻟﻤﻔﺮوﺿﺔ وﺗﻮ ﺑﯿﺘﻮﯾﻨﺞ‬
‫اﻻﺷﻌﺔ اﻟﻤﻔﺮوﺿﺔ وﺗﻮ ﺑﺎﯾﺖ وﻧﺞ وﺑﺎﻧﻮراﻣﺎ‬

✅✅
‫وﺧﯿﺎرﯾﻦ ﻛﻤﺎن ﻧﺎﺳﯿﺎه‬
Bite and periapical

‫**ﻟﻮ ﻟﺴﻪ اﻻﺳﻨﺎن ﻟﺒﯿﻨﻪ‬


occ 2
bite wing 2
And 4 peri apical

‫ ﻣﺎﯾﺎﺛﺮ ع اﻟﻠﺜﺔ واﻻﺳﻨﺎن‬- - Sjogren syndrome

✅✅
‫ﯾﺴﺒﺐ رووت ﻛﺎرﯾﺰ وﻣﺎﯾﺎﺛﺮ ع اﻟﻠﺜﺔ‬-
‫ﯾﻌﻤﻞ ﺗﺄﺛﯿﺮ ﺑﺎﻟﻠﺜﺔ ودﻧﺘﻞ ﻛﺎرﯾﺰ‬-

**Due to the xerostomia symptoms that commonly accompany Sjögren


disease, patients may present with dry and friable oral mucosa with a dry
and fissured tongue. Patients with Sjögren disease may commonly have
root, cervical, or incisal/cuspal tip dental caries, accumulation of plaque,
gingivitis, and/or periodontitis

‫ ﻣﺮﯾﺾ‬hepatitis ‫ ﻋﻤﻠﺘﻠﻪ‬surgery ‫ واﻟﺒﻠﯿﺪ ﺣﻄﯿﻨﺎه ﻓﻲ ﺑﻼﺳﺘﻚ ﺑﺎج واﺟﺎ اﻟﻌﺎﻣﻞ وﺣﺼﻠﻪ‬injury ‫ﺑﺴﺒﺒﻪ‬
‫اﯾﺶ اﻫﻤﻠﻨﺎ ﺑﺎﻻﻧﻔﻜﺸﻦ ﻛﻮﻧﺘﺮول؟‬

✅✅
‫ ﻫﺬه ﻣﻌﻨﻰ اﻟﺨﯿﺎر اﻧﻪ اﻫﻤﻠﻨﺎ اﻟﺮﻗﺎﺑﺔ‬..‫ﺳﻮﺑﺮﻓﯿﺠﻦ‬
Negligent supervision

‫ﻛﺎن ﻓﻲ ﺧﯿﺎرات داﯾﺮﻛﺖ وان داﯾﺮﻛﺖ‬


Heavy intermittent force cause‫؟‬
—Bone necrosis

✅✅
—Frontal resorption
—Undermined resorption
‫اﯾﻮة واﻟﻔﺮوﻧﺘﺎل رزورﺑﺸﻦ ﺑﺴﺒﺐ ‪light continuous firce‬‬

‫✅✅‬
‫?‪Systemic disease associated with gagging‬‬
‫‪Adenoid‬‬

‫اﯾﺶ ﻣﻤﻜﻦ ﺗﺸﻮﻓﻲ ﻋﻨﺪو؟ ‪ multiple sclerosis‬اﺟﺎ ﺑﯿﺸﻨﺖ ﻋﻨﺪو‬

‫✅✅‬
‫ﺗﺮﯾﺰﻣﺲ‬
‫ﻓﻲ ﻛﺎن ﺧﯿﺎر ع ﻣﺸﺎﻛﻞ ﺑﺎﻟﺘﻲ ام ﺟﻲ ﺑﺎﯾﻼﺗﺮاﻟﻲ‬
‫ اﺟﺎ ﺑﯿﺸﻨﺖ ﻋﻨﺪه‬hypertension ‫ وﯾﺴﺘﺨﺪم‬tongue ‫ﻣﺪري اﯾﺶ ﻧﺴﯿﺖ اﺳﻢ اﻟﺠﻬﺎز اﻟﻤﻬﻢ اﻧﻪ ﺟﻬﺎز ﯾﺤﻄﻪ‬
‫ﻓﻮق ﻟﺴﺎﻧﻪ‬
‫اﯾﺶ ﺑﯿﺴﺒﺐ ﻋﻠﯿﻪ؟‬

✅✅
Sialorrhea
Sleap apnea
‫وﺧﯿﺎرات ﻧﺴﯿﺘﻬﻢ‬

An old female pt is hypertensive...


Suffering from sleep apnea came for dental issue (‫)ﻣﺶ ﻓﺎﻛﺮة‬, mentioned that

✅✅
she has tongue holding or retaining aplliclance. Reason for its wearing
Sleep apnea
Tongue thrust

🏼
‫☝ﺑﯿﺸﺒﻪ اﻟﺴﺆال ده‬
‫ ﻣﺮﯾﺾ ﻋﻨﺪه‬edema in lower limb ✳✳‫ وﯾﺴﺘﺨﺪم‬diuretic ‫ وﻋﻨﺪه‬short breath ‫وﯾﻨﺎم ﺗﺤﺖ‬
‫ﻣﺨﺪﺗﯿﻦ‬

✅✅
‫?وﺳﯿﻨﺎرﯾﻮ ﻃﻮﯾﻞ‬
Congestive heart failure
MI
Renal failure
‫؟؟‪ which prevent complication in future‬ﻟﻼﻃﻔﺎل ‪ material‬اﺟﺎ اﯾﺶ اﺣﺴﻦ‬
‫ﺣﯿﺮﻧﻲ ﺟﺎﯾﺐ‬
‫اﻣﻠﺠﻢ‬
‫ﻛﻮﻣﺒﺰﯾﺖ‬

‫✅✅‬
‫ﺟﻼس اﯾﻨﻮﻣﺮ‬
‫ا ﺳﺘﻨﻠﺲ اﺳﺘﯿﻞ‬

‫‪✳✳Metalic sound‬‬
‫ﺟﺎﯾﺐ اﻻﺗﺮال ﻟﻮﻛﺴﯿﺸﻦ‬
‫و اﻧﺘﺮوﺟﻦ‬

‫✅✅‬
‫ﺟﺎب ﺳﻮال ﻛﯿﻒ اﻗﯿﺲ اﻟﺜﯿﻜﻨﺲ ﺣﻖ اﻟﺒﻮن‬
‫ﻓﻲ ﺳﻲ ﺑﻲ ﺳﻲ ﺗﻲ‬
‫‪ bone sounding‬وﻓﻲ ﺣﺎﺟﺔ‬
‫ﺟﺎب ﺻﻮرة اﻟﺒﺎﻧﺪ ﺑﻮﺷﺮ وﻻﯾﺶ ﯾﺴﺘﺨﺪم‬
‫وﺟﺎب ﺻﻮرة ﻧﯿﺒﺮس ﺑﺮوب وﻻﯾﺶ ﯾﺴﺘﺨﺪم‬
‫✔🍃‬
‫🍃‬
‫‪*Band pusher* > initial placment of band by dentist....‬‬
‫‪*Band seater* > final placment of band by patient...‬‬

‫ﻛﻤﻠﺖ ﺷﻐﻞ ﻟﺒﯿﺪ وﻋﺎوز ﺗﻤﺪﺣﻮ ﺑﻮزﺗﯿﻒ ري اﻧﻔﻮرﺳﻤﻨﺖ✅✅‬


‫ﺑﯿﺸﻨﺖ ﻓﺎﻗﺪ اﺗﻨﯿﻦ ﺳﺘﻨﺘﺮال وواﺣﺪ ﻻﺗﺮال ‪ 21-12-11‬اﯾﺶ ﺑﺘﺎﺧﺬ ﻣﻦ اﺑﺘﻤﻨﺖ؟‬

‫✅✅‬
‫اﻣﺎ ﺗﺎﺧﺬ اﻻﺗﺮال وﻛﻨﺎﯾﻦ‬
‫او ﻻﺗﺮال وﻛﻨﺎﯾﻦ وﺑﺮي‬
‫وﺟﺎﯾﺐ ﺧﯿﺎر ﺗﺨﻠﻊ اﻻﺗﺮال وﺷﻜﯿﺖ ﻓﯿﻪ ﻟﻠﻤﻨﻈﺮ اﻟﺤﻤﺎﻟﻲ‬

‫ﺟﺎب ﻣﻮﻟﺮ ﻋﻨﺪه اﻧﺪو ﻣﻦ ارﺑﻊ ﺳﻨﻮات وﺻﻮرة اﻻﻧﺪو ﺑﺮﻓﻜﺖ ﺟﺪا‬

‫✅✅‬
‫وذﻛﺮ اﻧﻪ ﻣﺎﻓﯿﺶ ﺑﻮﻛﺖ وﻻ ﺑﻠﯿﺪﻧﺞ ﻟﻜﻦ ﺗﻨﺪر ﺗﻮ ﺑﺮﻛﺸﻦ وﺑﺎﻻﺷﻌﺔ اﺑﯿﻜﺎل رادﯾﻮﻟﺴﻨﺴﻲ ﺣﺠﻤﻬﺎ ﻣﺶ ﻛﺒﯿﺮ ﯾﻌﻨﻲ وﺳﻂ‬
‫اﺧﺘﺮت ﺳﺮﯾﺠﻜﺎل روت ﺗﺮﯾﺘﻤﻨﺖ‬
‫ﻛﺎن ﻓﻲ ﻧﻦ ﺳﺮﺟﯿﻜﺎل رﯾﺘﺮﺗﻤﻨﺖ‬
‫اﻛﺴﺘﺮاﻛﺸﻦ‬

‫✅✅‬
‫اﻻﻧﺘﺮﻧﺎل رﯾﺰورﺑﺸﻦ ﻋﺒﺎرة ﻋﻦ?‬
‫رادﯾﻮﻟﺴﻨﺴﻲ اﻧﺘﺮﻧﺎﻟﻲ اﻧﺴﺐ ﺧﯿﺎر‬

‫اﻟﺒﻮﻟﻲ ﻣﯿﺜﯿﻞ ﻣﯿﺜﺎ اﻣﻜﺮﯾﻠﯿﺖ✅✅‬


‫?‪contraincation with pupl vital‬‬

‫‪patient with high palatal vault, posterior cross bite, pseudomembranous .40‬‬

‫✅✅✅‬
‫‪:candidiasis xerostomia and another 2 symptoms‬‬
‫‪asthma -‬‬

‫✅✅‬
‫ﻛﻢ اﻻﺳﺒﻠﻨﺖ ﻟﻼﻧﺘﺮوﺟﻦ اﻻﻛﺜﺮ ﻣﻦ ‪7mm‬‬
‫اذا اﻛﺜﺮ ﻣﻦ ‪ ٧‬ﺑﯿﻜﻮن ﻧﺮﺟﻌﻬﺎ ﻟﻤﻮﻗﻌﻬﺎ ﺟﺮاﺣﯿﺎ ﺑﯿﻜﻮن ‪ 4‬اﺳﺎﺑﯿﻊ ﺣﺴﺐ اﻟﻤﻜﺘﻮب‬
‫✳اﺟﺎ اﻧﻪ اﻻﻧﺎﻣﻞ ﻋﻨﺪه ﺗﺼﺒﻎ ﻛﺎن ﺑﺎﻟﺒﺪاﯾﺔ ‪ white‬وﺑﻌﺪﯾﻦ ﺗﺤﻮل ‪yellow brown‬‬
‫اﻣﻠﻮﺟﻨﺴﯿﺲ اﻣﺒﺮﻓﻜﺘﺎ‬
‫او دﻧﺘﯿﻨﻮﺟﻨﺴﺲ اﻣﺒﺮﻓﻜﺘﺎ‬
‫او اﻧﺎﻣﻞ ﻫﯿﺒﻮﺑﻼزﯾﺎ‬
‫او ﻓﻠﻮروﺳﯿﯿﺲ✳✳‬

‫ﻋﻨﺪه اﺳﻜﻠﺘﺎل ودﻧﺘﺎل ﻛﻼس ﺗﺮي وﻫﺎﯾﺒﻮﺑﻼﺳﺘﯿﻚ ﻣﺎﻛﺰﻻ اﯾﺶ اﻻﺑﻼﯾﻨﺎس اﻟﻲ ﺗﺴﺘﺨﺪﻣﻪ؟‬
‫ﻣﺎوث ﺟﺎرد‬
‫ﻫﯿﺪﺟﯿﺮ‬
‫✅✅‬
‫ﺗﻮﯾﻦ ﺑﻠﻮك‬
‫ﻓﯿﺲ ﻣﺎﺳﻚ‬

‫ﺟﺎب اﻟﺴﻮال ﺣﻖ اﻟﺴﻦ اﻟﻲ ﻣﻘﺎﺑﻞ ﻟﻼﻣﺒﻼﻧﺖ وﻋﺎﻣﻞ ﺳﻮﺑﺮ ارﺑﺸﻦ ﻛﺎن ﻓﻲ ﺧﯿﺎر اﻻﻧﺪو واﻟﻜﺮاون✅‬
‫اﺳﺌﻠﺔ ﻛﺜﯿﺮ ع اﻟﺴﯿﺮاﻣﯿﻚ و ‪pfm‬‬
‫اﻟﻮرﻛﻨﺞ واﻟﻨﻦ ورﻛﻨﺢ واﻟﺒﺮوﺗﺮوﺳﯿﻒ‬
?‫وﻛﯿﻒ ﺗﻌﻤﻞ ﻛﻮرﻛﺸﻦ ﻟﻤﺎ ﻓﻲ اﻧﺘﺮﻓﺮﻧﺲ‬

. Centric interference = reduction of mesial U and distal L

. Protrusive interference = reduction of distal U and mesial L

. Working side interference = reduction of buccle U and lingual L

.Nonworking side interference = reduction of lingual U and buccal L

Balancing interference = just buccle L

‫ ف ﻋﻤﻮﻣﺎ اﻟﺠﻮاب‬.. ‫ ﻣﻊ ﺑﻌﺾ ﻧﺴﻮي ﻟﻬﻢ ﻗﺮاﯾﻨﺪ ﻟﻠﻜﺴﺐ ﺗﯿﺐ‬centric ‫ ﻣﻊ‬eccentric ‫" ﻟﻤﺎ ﺗﺼﯿﺮ اﻟﻤﺸﻜﻠﺔ‬
‫اﻟﻤﺘﻔﻖ ﻋﻠﯿﻪ ﻫﻮ اﻻﻛﺴﻨﺘﺮك‬
‫اﻣﺎ ﻟﻤﺎ ﺗﻜﻮن اﻟﻤﺸﻜﻠﺔ ﻫﻲ اﻟﺴﻨﺘﺮك! ﻓﻘﻂ ﻣﺎﺗﻠﻌﺐ اﺑﺪا ﺑﺎﻟﻜﺴﺐ‬
‫ ﻟﻠﺴﻦ اﻟﻤﻘﺎﺑﻞ‬deepen of fossa ‫وﺗﺴﻮي ﺑﺲ‬

‫ اﻧﻪ ﻧﻨﻘﺺ‬correction ‫ ﺻﺢ ﻧﻌﻤﻞ‬protrusion ‫ﺟﺎﯾﺐ ﺳﻮال اﻧﻪ ﻓﻲ اﻧﺘﺮﻓﻨﺲ ﻟﻤﺎ ﯾﻌﻤﻞ‬


Distal incline if upper
?And mesial incline of lower
‫ﻫﻮ ﻣﺶ ﺟﺎﯾﺐ ﻛﺪا‬
‫ﺟﺎب اﻟﺨﯿﺎرات‬
Buccal maxilla
Buccal mandible
Lingual maxilla
Lingual mandibula

✅✅
?‫اﻟﻤﺮﯾﺾ اﻟﻲ ﻋﻨﺪه ﻛﻼس ﺗﻮ وﻛﺮاوﻧﺪﻧﺞ ﻻور اﯾﺶ ﺗﺨﻠﻊ‬
Upper4 lower 5
‫اﻟﺴﺆال ﺣﻖ ﺗﺤﻀﯿﺮ اﻟﻜﺒﯿﺮ وﺣﺸﻮ اﻟﺼﻐﯿﺮ‬
‫ﻟﻤﻦ ﯾﻜﻮن ﻋﻨﺪك ﺣﺸﻮﺗﯿﻦ‬

✅✅
‫ﺗﺒﺪي ﺑﺮﯾﺒﺮﯾﺸﻦ ﺑﺎﻟﻜﺒﯿﺮ‬
‫و ﻟﻤﻦ ﺗﺤﺸﻲ اﻟﺤﺸﻮة ﺗﺒﺪي ﺑﺎﻟﺼﻐﯿﺮ‬

✅✅‫اﻟﻔﺎﯾﺒﺮﺗﻨﺞ ﻻﯾﻦ ﺗﻲ ﺑﺮﻧﯿﺸﺮ‬


Reverse smoking?
Definition. Stomatitis nicotina (known as smoker's palate, smoker's

✅✅
keratosis, nicotinic stomatitis, stomatitis palatini, leukokeratosis nicotina
palate) is a diffuse white lesion covering most of the hard palate

‫ ﻣﺮﯾﺾ ﻋﻨﺪه‬hiv✳✳
‫ﺗﺸﺘﻐﻠﻪ‬
‫وﻻ ﺳﺎﻧﻲ ﺗﺤﻮﻟﻪ ﻟﻼﻧﻔﻜﺸﺲ ﺳﻨﺘﺮ‬

‫ اﻛﺲ رﯾﻪ اﺳﺌﻠﺔ ﺗﺨﺺ‬open contact or open margin


‫ وﺟﺖ ﻛﺜﯿﯿﺮ ﯾﺤﯿﺐ اﺷﻌﺔ وﻓﯿﻪ‬space ‫ﻣﺎﺑﯿﻦ اﻟﺤﺸﻮات ﯾﻘﻮﻟﻚ اﻟﺪﻓﻜﺖ ﺑﺴﺒﺐ اوﺑﻦ ﻛﻮﻧﺘﺎﻛﺖ وﻻ اﻟﺪﻓﻜﺖ ﺑﺴﺒﺐ‬
‫اوﺑﻦ ﻣﺎرﺟﻦ‬

**open contact ➡ between tow restoratio


Open margin ➡ between tooth and crown

14. A patient presented to the clinic for regular checkup (see image)
bitewing x-ray shows the crown on tooth #46 has open margin on both
mesial and distal (see image). Which of the following the most possible

✅✅
reason for open margin? A. Expansion of the core
B. Shrinkage of the casting
C. Over tapered preparation
D. Obstructed path of insertion
‫✅✅‬
‫ﺳﻮال اﻛﺲ رﯾﻪ ﻓﻲ اﺑﯿﻜﺎل اﻛﺴﺘﺮوﺟﻦ ﻛﺒﯿﯿﯿﺮ ﻟﻠﺠﻮﺗﺎ ﺧﺎرج اﻟﺬروة اﯾﺶ اﻟﺴﺒﺐ؟‬
‫‪Lack of apical stop‬‬
‫ﻓﻲ دﯾﺴﺘﻮ اﻧﺴﺎﯾﺰل ﻣﻜﺴﻮر واﻻﺑﯿﺎل ﻣﺘﺼﺒﻎ اﯾﺶ ﺗﻌﻤﻞ‬

✅✅
‫ﻓﯿﻨﯿﺮ‬
‫ﻓﻞ ﻛﺮاون‬

‫اﻟﺠﺮﯾﺴﻲ واﻟﯿﻮﻧﻔﺮﺳﻞ ﺟﺖ ﻛﻞ اﻻرﻗﺎم اﻟﻲ ﺗﺨﺼﻬﻢ‬

? treatment of dentigerous cyst


extract the tooth and the cyst-

✅✅
remove cyst only-
marsupilization-

‫ﻣﺮﻓﻘﺔ ﺑﺼﻮرة اﺷﻌﺔ ﺛﯿﺮد ﻣﻮﻟﺮ ﺑﺎرﺷﯿﺎل ارﺑﺸﻦ وﺳﯿﺴﺖ ﻛﺒﯿﯿﺮة‬

water line 20-30 seconds ✅✅

✳✳A 27 year - old man came to the clinic with intermittent severe pain in
tooth # 46. The pain lasts more than 3 minutes after cold drink in addition f
pain on biting. During clinic examination, tooth # 46 respond to cold test
with severe pain that lingers for more than three seconds (see report).
Radiograph: Showed a deep carious lesion and thickening of PDL space.
Slight tenderness to percussion was revealed which of the following is the
definitive diagnosis of tooth # 46?
A Symptomatic irreversible pulpits with normal apical tissues
B. Symptomatic irreversible pulpits with chronic apical abscess
C. Symptomatic irreversible pulpits with asymptomatic apical periodontitis
D. Symptomatic irreversible pulpits with symptomatic apical periodontitis

‫ اﺟﺎ دا اﻟﺴﻮال اﻟﺨﯿﺎر‬b ‫ و‬c ‫ﺗﻤﺎﻣﺎ ﺗﻤﺎﻣﺎ ﻏﻠﻂ‬


D ‫ﺷﻜﯿﺖ ﻻﻧﻪ ﺗﻨﺪر اوف ﺑﺮﻛﺸﻦ‬
‫‪ a‬وﻧﻲ ﺣﻄﯿﺘﻮ ‪ a‬د‪ .‬روﯾﺪا ﺳﺎﻟﺖ اﺧﺼﺎﺋﻲ وﺟﺎوﺑﻬﺎ‬

‫‪Dr.Afnan‬‬
‫اذﻛﺮوﻧﻲ ﺑﺪﻋﻮة‬
‫ﻣﻨﺎﻗﺸﺔ ‪ ١٤‬ﻣﺎرس‬

‫‪-١‬اﻟﺒﻜﺘﺮﯾﺎ اﻟﻠﻲ ﻓﻲ ‪C. P‬‬


‫اﻟﺒﻼزﻣﺎ ﻓﻲ اي ﻣﺮﺣﻠﻪ‬-٢
1st pmn

✅✅✅
2nd lymphocytes
3rd plasma

✅✅
ANUG ‫ ﻣﻜﺸﻮف اي ﻣﺮﺣﻠﺔ ﻣﻦ‬bone ‫ ال‬.٣
6
‫اﻟﺒﻮن ﻗﺮاﻓﺖ اﻟﺒﺘﺎﺧﺬه ﻣﻦ اﻟﻤﺮﯾﺾ ﻧﻔﺴﻪ اﯾﺶ اﺳﻤﻪ‬-٤
An autograft is a bone or tissue that is transferred from one spot to another
on the patient's body. It is often thought of as the “gold standard” in bone

✅✅
grafting because of its reliability. Its high success rate is due to the fact
that it is living tissue and thus its cells are kept intact
✅✅
5.mechanism of metronidazole
Inhibit DNa

Mechanism of Action

Metronidazole diffuses into the organism, inhibits protein synthesis by


interacting with DNA and causing a loss of helical DNA structure and
strand breakage. Therefore, it causes cell death in susceptible
✅✅
‫ اﻓﻀﻞ روت اﯾﺶ؟‬٦ ‫ﺑﻮﺳﺖ ﻛﻮر ﻓﻲ اﻻﺑﺮ‬-٦
Palatal

‫ﺗﻌﺮﯾﻒ اﻟﻤﺎﺳﺘﺮ ﻓﺎﯾﻞ‬-٧

✅✅
enlarging the apical portion of the root canal system three sizes larger than
the first file that bound at working length

the largest file reaching apical foramen✅✅

‫ﻋﻨﺪه ﻻرج ﻓﺴﺘﺒﯿﻮل اﻓﻀﻞ ﻧﻮع ﻣﯿﺠﺮ ﻛﻮﻧﻜﺘﺮ‬-٨


✅✅
Lingual bar
‫✅✅‬
‫‪-٩‬ﻛﺮاون ﻧﺎﻗﺺ ‪٣‬ﻣﻢ اﯾﺶ اﻟﺤﻞ ﺗﺮﺟﻌﻪ اﻻب ﺗﺎﺧﺬ ﻣﻘﺎس ﺟﺪﯾﺪ ﺗﻤﻞ اﻟﻔﺮاغ ﺑﺤﺸﻮه اﯾﺶ؟‬
‫‪Remake‬‬
xerostomia ‫ اﯾﺶ ﻧﻌﻄﯿﻪ دواء‬.١٠

✅✅✅
pilocarine (Salagen) or cevimeline (Evoxac) to stimulate saliva production.

**Pilocarpine is in a class of medications called cholinergic agonists. It


works by increasing the amount of saliva in the mouth.

‫ﺻﻮره ﻟﻠﻬﺎﻧﺪ رﺳﺖ )ﻛﺮوس ارش‬-١١


Cross arch : The finger rest is established on tooth surfaces of the teeth on

✅✅✅
the other side of same arch which is being worked on. Opposite
arch
‫‪-١٢‬اﻟﺮاس اﻟﻄﻮﯾﻞ واﻟﺮاس اﻟﻘﺼﯿﺮ اﯾﺶ اﺳﻤﻬﻢ?‬
‫✅✅‬
‫‪Declilocephalic‬‬

‫‪-١٣‬ﻛﯿﺲ اﻣﺮﺟﻨﺴﻲ ﻋﻨﺪه ﻛﺴﺮ ﺑﺎﻟﻮﺟﻪ واﻟﻔﻚ ﻣﺎﯾﻘﺪر ﯾﺘﻨﻔﺲ اﯾﺶ ﺗﺴﻮي اول ﺷﻲ؟‬
Maintain airway✅✅✅

✅✅
?‫ اﯾﻪ اﻓﻀﻞ ﺣﺸﻮه‬٤ ‫ﻛﻼس‬-١٤
Hiberd and microfill
?AH plas ‫ ﻋﻦ ال‬.١٥

✅✅
AH plus sealer working time : (epoxy sealer past )
• 4 hours

4 H working time
8 H setting time

**
AH26 working:15 h ,
setting time :24-36 H
‫)‪-١٦‬ﺳﺒﺐ اﻻﻧﺘﯿﺮﻧﻞ رﯾﺴﻮﺑﺸﻦ )ﺣﺮﻛﺖ اﻻﺳﻨﺎن ﺑﺴﺮﻋﻪ اورﺛﻮ وﻻ اﻧﻔﻜﺸﻦ ﻟﻞ ﻛﺮوﻧﻞ ﺑﻠﺐ وﻻ اﯾﻪ‬
‫‪ .١٧‬ﻛﯿﺲ ‪ RCT‬ﻣﺘﺄﻟﻢ و اﻟﺮوت ﻛﯿﺮف اﯾﺶ ﺗﺴﻮي‬
✅✅
‫ﺗﺤﻮﻟﻪ ﻻﺧﺼﺎﺋﻲ وﻻ ﻣﺎ ﺗﺴﻮي ﺷﻲ وﻻ ﻣﺴﻜﻦ وﻻ ﺑﻠﺒﺘﻮﻣﻲ‬
Pulpetomy then refer

✳✳‫ ﺳﻮﯾﺖ ﺑﯿﺮﻓﯿﺮﯾﺸﻦ ﻣﺘﻰ ﺗﻌﺎﻟﺞ اﻟﺒﯿﺮﻓﯿﺮﯾﺸﻦ‬RCT ‫اﺛﻨﺎء ال‬.١٨

which of the following is the generally preferred time for repairing the .84
defect if a cervical root perforation occurs during incomplaete root canal
?treatment
A. Immediately after obturation

✅✅
B. After cleaning and shaping is complete but before obturation
C. Immediately, and before proceeding with further preparation
D. After an appropriate recall period, to assess the status of the tissues

✅✅
١٩- safest LA for pedo?
2%lidocanin

‫ اﺳﺎﻟﻪ ﻋﻦ اﻧﻮاع اﻟﺒﺮوب‬٣ .٢٠


✅✅
‫؟‬GB ‫ ﻓﻲ ال‬GB ‫ ﻧﺴﺒﺔ ال‬.٢١
20%

Gutta-percha endodontic filling points were found to contain approximately


20% gutta-percha (matrix), 66% zinc oxide (filler), 11% heavy metal sulfates
(radiopacifier), and 3% waxes and/or resins (plasticizer). The mechanical
properties were indicative of a partially crystalline viscoelastic polymeric
material.

‫اﻻﺧﺼﺎﺋﻲ ﻛﺘﺐ ﻟﻠﻤﻤﺮﺿﻪ وﻛﺬا اﯾﺶ ﯾﻌﻄﻮ اﻟﻤﺮﯾﺾ وراح واﻟﻤﺮﯾﺾ ﻋﻨﺪه ﻣﺮض ﻣﻌﺪي اﻟﺪﻛﺘﻮر‬-٢١

✅✅
‫ارﺗﻜﺐ ذﻧﺐ ﺑﺤﻖ ﻣﯿﻦ؟‬
Community

6‫ﺻﻮرة ل ﻛﺮﺑﻠﻲ ﻛﺴﺐ ﻓﻲ اﻻﺑﺮ ﺑﻠﺖ‬-٢٣


‫✅✅‬
‫‪-٢٤‬ﻣﺮﯾﺾ ﯾﺤﻂ ‪٣‬ﻣﺨﺪات ﺗﺤﺖ راﺳﻪ وﻋﻨﺪه ﺗﻮرم ﺑﺮﺟﻠﻪ وﻣﻊ اﻟﻤﺸﻲ اﻟﻘﻠﯿﻞ ﯾﺘﻌﺐ اﯾﺶ ﻋﻨﺪه؟‬
‫‪Congestive heart failure‬‬
‫✅✅‬
‫‪-٢٥‬ﻃﻔﻞ ﻋﻤﺮه ‪٥-٤‬ﺳﻨﻮات ﻛﻼس ‪ (3‬ﻓﻜﺸﻨﻞ اﺑﻠﯿﻨﺲ وﻻ ﺗﻨﺘﻈﺮ ع ﺑﺎل ﯾﻜﺒﺮ وﻻ اﯾﺶ‬
‫‪Waiting for mixed dentation‬‬
✅✅
‫ﺑﯿﺸﻦ ﻋﻨﺪه اﻧﺪوﻛﺮداﯾﺘﺲ اﯾﺶ ﺗﺴﻮﯾﻠﻪ ﻗﺒﻞ اﻟﺠﺮاﺣﺔ‬-٢٦
Prophylactic antibiotics 2gm

‫ ﺑﻠﻮك‬NP ‫ ﺻﻮره ل‬.٢٧

‫ﻣﺮﯾﺾ ﺟﺎء ﻟﻼورﺛﻮ واﻻورل ﻫﺎﯾﺠﯿﻦ ﺳﻲء ﯾﻮم ﺳﺎﻟﻪ اﻟﺪﻛﺘﻮر ﻋﻦ اﻟﺘﻔﺮﯾﺶ اﯾﺶ ﺟﺎوب ﻋﻠﻰ اﻟﺪﻛﺘﻮر‬-٢٨✳✳

😅😬😂👇👇👇‫ﻣﺘﻞ ﻫﺎﻟﻨﻤﻂ‬
aque was 10% and bleeding was 60 % this indicate : a- patient knows how
to brush and is brushing

✅✅
b- patient knows how to brush but not brushing
c- patient doesn't know how to brush but trying
d- patient doesn't know how to brush and not trying
:plaque was 40 % and bleeding was 20 % in recall visit this indicates -3

✅✅
a- patient knows how to brush and is brushing
b- patient knows how to brush but not brushing
c- patient doesn't know how to brush but trying
d- patient doesn't know how to brush an

..‫ ﺑﺴﯿﻂ ﻣﻌﻨﺎﻫﺎ ﺑﯿﺤﺎول ﯾﻨﻈﻒ ﺑﺲ ﻣﺎﯾﻌﺮف اﻟﻄﺮﯾﻘﺔ اﻟﺼﺤﯿﺤﺔ‬plaqu‫ ﻛﺒﯿﺮة وال‬bleeding‫**اذا ﻧﺴﺒﺔ ال‬

..‫ ﻛﺘﯿﺮة ﻣﻌﻨﺎﻫﺎ ﻣﺎﺑﯿﻨﻈﻒ اﺳﻨﺎﻧﻮ‬plaque‫واذا ﻛﺎﻧﺖ ﻧﺴﺒﺔ ال‬

#zygoma ‫ﺻﻮرة اﺷﻌﻪ‬.٢٩


‫ﻛﺴﺮ ﻓﻲ ﻋﻈﻤﺔ اﻟﺰﯾﻘﻮﻣﺎ وم ﯾﻘﺪر ﯾﻔﺘﺢ ﻓﻤﻪ وﻋﻨﺪه ﻣﯿﻼن ﻓﻲ اﻟﻔﻚ‬

Submento vertex : best for zygomatic arch fractures ✅✅✅

✳✳‫اﺛﻨﺎء اﺧﺬ اﻟﻤﻘﺎس اول ﻣﺎ اﻟﻄﺒﯿﺐ ﺟﺎء ﺑﯿﺎﺧﺬ اﻟﻤﻘﺎس اﻟﻤﺮﯾﺾ رﺟﻊ ﻟﻮرا ﻣﻤﺒﺎﺷﺮه اﯾﺶ اﻟﺴﺒﺐ ؟‬-٣٠
Polysulfide > unpleasnt odor and taste
‫‪-٣١‬ﻣﺮﯾﻀﻪ ‪٢٢‬ﺳﻨﻪ ﻋﻨﺪﻫﺎ ﻓﺸﻞ ﻛﻠﻮي ﺑﻌﺪ اﻟﻐﺴﯿﻞ ﺟﺎت ﻣﻊ اﻟﻤﻤﺮﺿﻪ وﺗﺒﻲ ﺗﺴﻮي ﻣﻌﺎﻟﺠﺎت اﻟﺪﻛﺘﻮر ﯾﺎﺧﺬ‬

‫✅✅‬
‫اﻟﻤﻮاﻓﻘﻪ ﻣﻦ ﻣﯿﻦ؟‬
‫ﻣﻦ اﻟﻤﺮﯾﻀﺔ‬

‫✅✅‬
‫‪-٣٢‬ﺑﻌﺪ اﻻﺳﻜﯿﻠﻨﺞ اﯾﺶ اﻟﺒﺮوب اﻟﻠﻲ ﻧﺴﺘﺨﺪﻣﻪ ﻟﻨﺸﻮف ﻣﻠﻤﺲ ﺳﻄﺢ اﻟﺠﺬر؟‬
‫‪Expoler‬‬

‫‪-٣٣‬ﺻﻮرﺗﯿﻦ ﻟﻜﻼس ‪ 3-2‬ﻓﻲ اﻟﺴﯿﻔﺎﻟﻮﻣﺘﺮ‬

‫✅✅‬
‫‪-٣٤‬ﻣﺮﯾﺾ ﺧﻠﻊ اﻟﻠﻮر ‪ ٦‬ﻟﻔﺖ ﻣﻌﺎﻟﻮﻗﺖ اذا م ﻋﻮﺿﻪ اﯾﺶ راح ﯾﺼﯿﺮ ؟‬
‫‪malaligement‬‬

‫✅✅‬
‫‪-٣٥‬اﻻم ﺣﺎﻣﻞ وﺗﺪﺧﻦ اﯾﺶ راح ﯾﺼﯿﺮ ﻟﻠﺠﻨﯿﻦ‬
‫‪Chance Cleft lip to baby‬‬

‫‪-٣٦‬اﺛﻨﺎء ﻗﺺ اﻟﺴﻠﻚ اﻟﺰاﺋﺪ ﻓﻲ اﻻورﺛﻮ اﻟﺤﻤﺎﯾﻪ ﻟﻠﻤﺮﯾﺾ اﯾﺶ ﻣﻮ ﻟﻠﺪﻛﺘﻮر✳✳‬


‫رﺑﺮ دام(‬
‫وﻻ ﺗﻠﺒﺲ اﻟﻤﺮﯾﺾ اﻟﻨﻈﺎرة ‪-‬‬
‫)وﻻ اﯾﺶ‬
‫اﻧﺎ اﺧﺘﺮت اﻟﻨﻈﺎره ﺑﺲ ﯾﻮم ﺳﺎﻟﺖ ﻗﺎﻟﻮ رﺑﺮ دام ﻋﺸﺎن اﻟﺴﻠﻚ اﻟﺰاﯾﺪ ﻣﺎ ﯾﺒﻠﻌﻪ اﻟﻤﺮﯾﺾ ﻟﻮ ﻗﺎل ﻟﻠﺪﻛﺘﻮر ﻧﻘﻮل ﻧﻈﺎره‬
‫‪-٣٧‬ﺻﻮرة ل اﻣﯿﻠﻮﺟﻨﯿﺴﺲ?‬

‫‪ ٤-٣٨‬اﺳﺎﻟﻪ ﻋﻦ اﻻﺛﯿﻜﺲ‬

‫اﻻﺛﻜﺲ‬
‫‪-١‬اﻻﺑﻮة‬
‫‪-٢‬اﻟﺼﺪق‬
‫‪ -٣‬ﻋﻠﻢ اﻻﺧﻼق‬
‫‪ -٤‬ﺣﻖ اﻟﺪﻛﺘﻮر ﯾﻌﺮف ﻋﻨﺪه ﻣﺮض ﻣﻌﺪي ﺑﺲ ﻛﺎن ﻣﺴﺘﻌﺠﻞ وم ﻛﻠﻢ اﻟﻤﻤﺮﺿﻪ وﻛﺬا‬

‫‪Dr.Afnan‬‬
‫اذﻛﺮوﻧﻲ ﺑﺪﻋﻮة‬
‫ مارس‬١٥ ‫مناقشة‬

1.crown calcification of permanent teeth complete after erruptin by:


A-6months

✅✅
B-12
c-24

**6 week intra utrine frist step of formation


6 months intra utrine frist of calcification
6 months baby life frist primary tooth erup
6 years of children life frist permenent teeth erup

✅✅
2.Ratio in nitrous oxide sedation
A-N2O 40% /70% . O2 60%/30%
B-N2O 90%/10% O2 10%/90%

3-The Amount of Xylitol for Child with 4 year Old :


A-2-5 mg\day
B-8 mg\day
✅✅C-3-8 mg\day

4-Patient with Sever Systemic Disease :


A-ASA 1
B-ASA 2

✅✅
C-ASA 4
D- ASA 3
✅✅
-The Optimal Amount of Fluoride in Drinking Water :
0.7-1.2ppm

-Primary Tooth After Trauma Was Discolored But With no Signs of Apical
Pathology What Should You

✅✅
do :
A-Follow Up
B-Extraction

-Primary Tooth After Trauma Was Discolored But With no Signs of Apical
Pathology What Should You

✅✅
do :
A-Follow Up
B-Extraction

✅✅
15-The concentration of APF :
A-1.23 %
B-5

28-Characteristic of Diabatic patient :

✅✅
A-Increase Infection , Periodontal Disease , Increase in Gingival Cervical
Fluid
B-Increase Infection , Periodontal Disease , Decrease in Gingival Cervical Fluid
‫ مش فاكرهم‬2 ‫كان فيه كمان‬

Innervation for Upper First Permanent Molar :


A-PSAN

✅✅
B-MSAN
C-PSAN & MSAN

24-Child Patient Came to Clinic With his Mother he Look Shy Stands Behind his

✅✅
Mother :
A-Timid
B-Defiant

42-Patient With Proximal Caries And Only Small Occlusal Caries What Is The
Best Restoration :
A-Composite

✅✅
B-Amalgam
C- compemer

-When You Do Pulpotomy And Bleeding Can’t Be Controlled What Should You

✅✅
Do :
A-Pulpectomy
B-Repeat Application of FC

-Best Word To Describe Paralysis Of Tounge :


A-Glossoptosis

✅✅
B-Macroglossia
C-Glossoplegia

Child 6 Years Old and with 0.6 Ppm of Fluoride What Is The Amount of

✅✅
Supplement He Should Take?
0,25

-Child Weighted 10Kg And After he Take 4 Carpule of LA Suddenly Arrhythmia

✅✅
Happen What is Cause :
A-LA Toxicity
B-Vasovagal Attack

-Pediatric Patient In His First Dental Visit To Pediatric Clinic What Is Greatest
Fear For Him :
A-Unknown ✅✅
B-Dentist
C-Needle

-Trigger Factor Of Sickle Cell Anemia?


The main pathology of the sickle cell syndrome is an abnormality of the
haemoglobin gene, the oxygen carrying protein of the bloodstream. The sickle
cell form of this gene is most commonly carried by the African population, with
approximately 8% of African Americans carrying the sicle cell disease gene.
‫✅✅‬
‫‪-At which age we can give child Aspirin:‬‬
‫‪A-16 y‬‬
‫‪B-12 y‬‬

‫‪ sedation‬سؤالين انك بتدي‬


‫و بيحصل اعراض‬
‫سؤال الطفل مش بيستحيب الي حاجه خالص‬
‫‪ repeted stimuli‬والسؤال التاني بيستحيب بعد‬

‫‪General anesthesia‬‬ ‫✅✅‬


Slob technique
,‫سؤالين بيستخدم ف ايه‬
‫وهو عباره عن ايه‬

✅✅
Same lingual opposite bucaal
Use impacted tooth

✅✅
Separation and identufication of superimposed canals
✅✅
-renal faliure pt
a-tooth paste without fluoride
b-toothpaste with fluoride
c-toorhpaste without fluoride,fluoride varnish
d-toothpaste with fluoride and f varnish

✅✅
-pt with hematopoietic liver dise. need liver transplant:
a-tooth paste without fluoride
b-toothpaste with fluoride
c-toorhpaste without fluoride,fluoride varnish
d-toothpaste with fluoride and f varnish
**Many studies have shown that exposed to high fluoride concentrations in


drinking water elevated the levels of renal and liver function enzymes in serum
and cause severe histological changes of the liver and kidneys

✅✳️
Pulse oximetry work by
Oxigenated blood absorb light or transmit light
Or de oxigenated blood absorb light or transmit

**Oximeters operate based on this principle of different absorption and light


emission of the T and R configurations. ...Oxygenated hemoglobin absorbs more
infrared light and allows more red light topass through. Deoxygenated
hemoglobin allows more infrared light to pass through and absorbs more red light

Mother bring the 4 years Child


And have sign of physical abuse and abcessed tooth
You give him antibiotc
‫اكدا دكتور اخترق ايه من االيثكس !؟‬

✅✅
Benifiance
‫الن الصح انه يعمل تقرير وهو كدا معملش الصح‬

Mta= mineral trioxide aggregate ✅✅

✅✅ ‫سنة والكناين ماطلعش‬١٣


Which situation need cbct

‫ ماطلعش‬8‫ سنه و‬17


‫ سنين وسنترال ما طلعتش‬7

✅✅✅
-Most impacted tooth
Lower third molar
**‫ يعني‬most impacted
Lower8
Lower 3

Most conginital missing


Wisdom
Lower 5
Upper 2
Upper 5

-Scorbutic gingivitis
Scorbutic gingivitis is associated with vitamin C deficiency and differs from the
type of gingivitis related to poor oral hygiene. The involvement is usually limited

✅✅
to marginal tissues and papillae. The child with scorbutic gingivitis may complain
of severe pain. Spontaneous hemorrhage may be evident.

-Dose of Amoxicillin for Patient with Infective carditis and will undergo for
invasive dental procedure
and he weighted 20 Kg :

✅✅
A-500 mg
B-1000 mg

✅✅١٠٠٠ = ٥٠ ‫ في‬٢٠ ‫*** نضرب‬


‫‪Dr.Afnan‬‬
‫اذكروني بدعوة‬
‫مناقشة ‪ ١٦‬مارس‬

‫‪⛔️١‬‬
‫بيشنت عندها اوبن بايت وقالها االورثو الوقت ‪ ١٨‬شهر وهي مستعجله‬
‫من ضمن الخيارات كراون لنثنق وال اكلوزل اجسمنت‬
‫بقية الخيارات كان تسوي بون دي كورتيكيشن‬

‫‪surgically‬‬ ‫✅✅‬
‫️⛔‬
‫‪ - ٢‬كان في مثل هذي الصورة بس شخص حقيقي ‪ injury upper lip‬و السؤال كم طبقة صار فيها‬
‫‪ injury‬او ال‪stage‬‬
3 layers ✅✅
‫‪⛔️٣‬‬
‫كان في مثل هذي الصورة و سأل عن وشو هذا و ان المريض متضايق منه مع االكل‬

‫✅✅‬
‫‪Mucocele‬‬
⛔️٤
‫في مثل هذي وشو هو وشو العالج؟‬

✅✅
Linear erythema _ fluconazole
‫او‬- ortho-veneer ‫في خيار‬ ⛔️
lateral‫ تقريبا مثل هذي الصورة و ب يعوض ال‬٥
FPD ‫ هذا الي اذكر و كان في‬ortho-important

Ortho implant ✅✅

٦⛔️Endodontitis + oral surgeon ‫ ياخذ القرار متى ممكن نساوي‬implant ?


oral surgeon ✅✅

⛔️
7 - A 4 year old pt presented in the clinic with her mother after injury to
her upper front teeth, x ray reveals mid root fracture in both 51 and 61,
there’s no mobility detected, management?

Observation ✅✅
⛔️
8 - Ratio( not percentage) of carbamide peroxide to hydrogen peroxide in

✅✅
bleaching?
1:3
H2o2 30%
Carbamide 10٪

⛔️
9 - A case of a failed (fallen out) laminated veneer of tooth 31... 2
pictures were provided in the question..clean surface NO cement on the
tooth and another pic for the veneer’s fitting surface there’s white cement (
cement on the veneer only)
What’s the cause for failure?

✅✅
Old saline
Improper etching of the porcaline
Predominantly dentinal substrate
Contamination of the porcelain surface

❇️‫لو السيمنت ع السنه مشكله ف الفنير‬


‫ لو السيمنت ماسك ف الفنير مشكله ف‬itch

⛔️
10 - Well controlled hypertensive patient with previous heart disease,
came for simple extraction and is taking anticoagulants

Request INR ✅✅
Give LA with epinephrine
Give LA without epinephrine
Reschedule appointment

11⛔️- A case about aplastic anemia


Normal MCV + Iess reticulocyte ✅✅

⛔️
✅✅
12 - Most common cause of FPD failure?
caries
⛔️
13 - how to ensure even thickness of the ceramic? (Options were mainly

✅✅
about lab tech.)
⛔️
14 - 3 unit of all ceramic bridge replacing #22 , fracture half of the incisal
third of 11, how to manage?

✅✅
Remake
- apply direct composite
Remove it and send it to lab
- I don’t remember the fourth option
⛔️
14 What cement has flouride flux? I - don’t remember the format of the

✅✅
question exactly.
GIC

⛔️
✅✅
15 - A case about osseous resective surgery, what’s the goal? ( perio)
Pocket reduction

16⛔️- Management of a child ingested 50 mg Fluoride✅✅


⛔️ -
17 ✅✅
Type of irrigation in open apex
⛔️
18 - patient has moderate pain days after complete endo treatment that’s
no non responsive to ibuprofen 600, what’s the drug of choice? 1- Combine
paracetamol 2- double the dose

Iboorufin 600+acetaminophen-1000mg ✅✅
‫‪⛔️19‬‬
‫✅✅‬
‫صوره القدم وشو المرض‬
‫‪Papillon lefevre‬‬
‫️⛔‬
‫✅✅‬
‫وجاني سوال االلسر الي في الفم واليد واالرجل ‪20‬‬
‫‪hand foot and mouth disease‬‬
‫‪⛔️21‬سؤال طفل نجرح في لسانه في صورة ايش العالج✅✅‬
⛔️
‫بيشنت تعب بعد اللوكل انسثيزيا ايش حصل‬ 21
Child Weighted 10Kg And After he Take 4 Carpule of LA Suddenly-

✅✅
: Arrhythmia Happen What is Cause
A-LA Toxicity
B-Vasovagal Attack
⛔️
22‫ ‏‬- The most common disease that is transmitted in the clinic through

✅✅
blood
HBV

23 ✳️✳️⛔️ - How to reduce torque forces on mesially tilted 37 abutment for


a bridge?

24 ✳️✳️ ⛔️
- When he worked for a patient who was pregnant with hepatitis,
what needed to be done by the doctor

25 ⛔️ What is the diameter of the probe?


✅✅
-‎ ‫كم قطر المسبر الحول سني‬
0.5

⛔️
✅✅
26‎ - ‫لما تقلع سكند موالر لبني والموالر الدائي بدو يطلع ازا ماحطيت حافظة مسافة شو بصير‬

✅✅
malocclusion
Loss of space

27- ⛔️‫مريض عم يحسن بطعم مو منيح بس السبب مو مرضي اله عالقة بالتاج‬
A sick person who improves the taste is not good, but the reason is not my

✅✅
disease has to do with the crown
loose abutment retainer
⛔️
✅✅
28 white lesion that forms the bottom of the mouth at the lingual frenulum
premalignant oral lesion
‫️⛔‬
‫✅✅‬
‫‪29‬‬ ‫لالملغم ومانو حاطط خيارة القوة او هيك‪ copper‬ليش بنحط‬
‫‪Coorosion protection‬‬

‫️⛔‬
‫✅✅‬
‫مريض ايدز شو بتعطيه‬ ‫‪-30‬‬
‫فلوكانزول‬
‫وال بريندوزون ؟‬

‫️⛔‬
‫✅✅‬
‫‪31‬‬ ‫‪-McCall festoon cause‬‬
‫‪Truma‬‬
⛔️

32 ⁃ least tooth to be impacted
Lateral

⛔️
33 -Posterior teeth which type of x ray
-For what
‫ السوال ناقص بس اذا علشان‬proximal caries
Periodental
‫ بيكون‬bit wing

‫ واذا عشان‬bone lose vertical bitewing

✅✅
‫واذا‬general bone lose opg

34⛔️⁃ Condylar rotate in degree✅✅


‫ن‬
**Incisal= 0
Lateral = 15
Protrusive = 30

⛔️
✅✅
35⁃ Limit heat during implant placement
47 for 30 sec

⛔️
36 Nurse when opening the pouch couldn’t see the indicator, indicator
should be :
A-in one corner

✅✅
B-in two corners
C-in the center
D-in two diff corner
⛔️
37 -Pt. came to clinc complain of mild pain after 2 days of amalgam
restoration and the pain subsided gradually what is your management?

✅✅‫نطمن المريض وبعدين اذا حاء في الم وفي اوكسيد نعيد الحشوه‬
⛔️
✅✅
38 -Tear drop appearance on x ray indicated for what?
Orbital fracture
‫**حسب االختيارات وتفاصيل السوال‬

⛔️
39 -Route of transmission of nosocomial infection?

✅✅
contact, droplet, air, water, food, or disease vector carrying and transmitting
an infectiouspathogen, or blood
⛔️
✅✅
40 -Best x ray to assess biological width?
Bitwing

Dr.Afnan
‫اذكروني بدعوة‬
‫ مارس‬١٧ ‫مناقشة‬

?Most common bacteria in water line-1

✅✅
Legionella pneumophila, Mycobacterium spp., Pseudomonas aeruginosa, and
Staphylococcus spp

🛑?Position of bacterial spore indicator-2


center ✅✅

.Exposed den.nal tubules in area of denuded cementum may serve as


communica.on pathways between the pulp and periodontal ligament.

Which of the following are the possible cause for the exposure of den.nal
tubules?
A . ver.cal root fractures

✅✅
B . post space prepara.on
C . developmental defects
D.den.nal hypersensi.vty

🛑Pt. came to clinc complain of mild pain after 2 days of amalgam restoration
and the pain subsided gradually what is your management !?

assure the patient only analgesic✅✅


🛑Material has adverse effect when put it directly on the exposed pulp !?
Amalgam

✅✅
Composite
ZOE
GIC
🛑Some doctors take the certificate without attending the course or *12 ???
lecture, this violation of what

verasity ✅✅
🛑Most common cause of pulpal infection?
most common cause of pulpitis is when bacteria irritate the dental pulp through

✅✅
an area of tooth decay, including dental caries. Other causes of pulpal
inflammation include: trauma or injury to a tooth

Bacterial ✅✅
🛑Difficulty of crown lengthening in lower second molar !?
Lingual nerve✅✅
mylohyoid ridge
Mental nerve

🛑Time of splinting for dentoalveolar fracture


4 weeks ✅✅

🛑pic of odontogenic infection and ask about cause which tooth !?


molars ✅✅
premolar
canine

🛑x-rays and ask about endo mischape

🛑Which virus can transmitted rapidly if the dentist remove his mask?
Tb✅
Hiv

HBC
🛑treatment of anterior open bite
‫ هنا ماكان في‬appliances

Intrusion of posterior teeth and extrusion of anterior teeth ✅


Extrusion of posterior and intrusion of anterior
‫باقي الخيارات م افتكرها بس مالها عالقه‬

🛑if the average length from gingival margin to bone crest is 3 mm , the sulcus
lengt will be?? In mm

✅✅
.5
1
2
1.5

3_2.04 bw= 0.96


‫مم‬١ ‫تقريبا‬

🛑sign of pathological occlusion or something like that


✳️✳️
The 7 sign and symptoms of occlusal disease are;

Pathological occlusal wear and fractures of teeth/restorations. Ratcliff S. ...

Cervical dentin hypersensitivity. ...

Tooth hypermobility. ...

Fremitus.

Abfractions. ...
Vertical bone loss or localized bone destruction (secondary to periodontal
disease). ...

Masticatory muscle or TMJ pain.

🛑best vitality test for tooth with deep amalgam restoration


Cavity test✅✅

🛑pt with sickl cell anemia, what is the most important thing you should do before
major surgery

✅✅
Prophylactic ab
Prepare platelets transfusion
‫م كان في اوكسجين ودي اقرب خيارات‬

🛑pic of papillon levefer syndrome -


PLS is characterized by periodontitis and palmoplantar keratoderma. The severe
destruction of periodontium results in loss of most primary teeth by the age of 4
and most permanent teeth by age 14. Hyperkeratosis of palms and soles of feet
appear in first few years of life
🛑first step in treatment of NUG
remove necrotic tissue
‫ما كان فيه واقرب شي باقي الخيارات ماليها عالقه‬


non surgical treatment

🛑shearing cusp of posterior cross bite?


UBLL✅✅
🛑class 1 malocclusion
✅✅
MB cusp of upper first molar occluded with MB groove of lower first molar

🛑prophaylactic antibiotic for MI pt


2 g amoxicillin 1 hr befor surgery✅✅

🛑management of CD pt with old aged appearance


Reline
Ramake

✳️✳️
Rabase

Advantages radial motion of rotary 🛑


🛑cuses of alginate distortion
🛑pic of non metallic ultrasonic tip uses
implant ✅✅

🛑pic of one wall bone defect and what it’s name


hemiseptal✅✅

🛑distance between papilla and labial surface of anterior teeth


8-10mm✅✅
🛑treatment of dry socket
Irregation+analgesic✅✅
🛑enamel pearls and why it’s considered as peridisposing factor of
periodontitis!?
If left untreated, an enamel pearl may cause gum and bone tissue destruction. It
may cause inflammation and periodontal pockets—gaps between the tooth and
gums where bacteria can collect—which jeopardize the health and longevity of

✅✅
the tooth involved

🛑acid eatch for GIC , acid eatching for tooth


Cavity etching before applying GIC is:

✅✅
1. Polyacrylic acid 10 seconds

🛑ethics >> autonomy, football player, definition of neglected child abuse


negleted child abuse ..

an egregious behaviour of caregivers that result in a deprivation of child of their

✅✅
basic needs including the failure to provide adequate suoervision , health care ,
clothing , or hiusing as well as other physical , emotional and safety needs

🛑
‫جاتني تقريبا نفس هذي الصور ف االختبار ويسأل عن التشخيص‬
Amelobastoma ✅
Osteosarcoma
Radicular cyst

**soap bubble appearance>> ameloblastoma

🛑
‫وهنا يسأل عن نوع الكسر او اسمه‬
Simple

✅✅
Cominuted
Compound
Green stick
‫💕‬
‫🏾🙏‬
‫‪Dr.Afnan‬‬
‫اذكروني بدعوة‬
‫ مارس‬١٨ ‫مناقشة‬

1. pain on upper molar ,normal response to cold and empty, no tenderness


on palpation or percussion ,pain started by brushing, touching or lying to
one side..?

Periodontal abcess

Trigeminal neuralgia ***

Pulpitis

2. a child with intermittent swelling and sinus tract related to lower


premolars area, teeth are caries free and perio tissues are healthy there’s
wide root canals with open apex whats the cause :

-Dene invagintus

- internal root resorption **

- occlusal trauma

3. pt on nifidipine complaints of gingival bleeding, gingvl enlargement and


probing depth 3 4mm.. management;

Ask the physician to change drug**

scaling and root planning

Oral hygiene instruction scaling


4. Periodontal complications associated with overhanging restoration ?

Plaque retention**

Amalgm tattoo

5. A 55 yr old man with deep stained molar, no softening.. what is the


management?

Pit and fissure sealant

Reassure the pt **

Preventive resin restoration

6. A 6 yr old boy, radiograph of midroot fracture of primary upper centrals


due to trauma mngmnt

Follow up

Remove both segments

Leave apical segment**

7. the abutment tooth of overdenture with post and core.. Detachment of


metal coping, otherwise it's ok..mngmnt?

Recementation of coping with gic

with resin cements***

8. bone grafting between 45 44 ,sufficient space interdental which flap

*Similar question
(19) A 42 - year - man presents to the Dental Clinic with penodontibs.

Clinical examination reveals a 3 - wall infrabony defect that needs


pramolars

bone grafbng mestal to mandibular second premolar. Suffieent Interdental

between first and second premolars was observed. '&ich of vs the

penodontal flap of ch0tce tn this case?

A, Partial thickness flap

B, Apically displaced flap

C Papilla preservation flap ***

O. Laterally positioned flap

9. root canal perforation material? (mta not in options)

Cavit

Bioceram***
10. component of LA responsible for anaphylactic reaction?

Methyl paraben**

Epinephrine
11. pt with history of dizziness, on previous LA

Change LA type **
Make pt comfortable

Extraction without LA

12. pt with MI hitory 3 yrs back on oral anticoagulants came for xtractn

Minitor inr

Refer to physician**

Do extraction

13. hypertension pt before extraction had dizziness, tachycardia ..

Pt missed morning drug

Hypoglycemia ***

Hyperglycemia

14. while inserting the denture pts mandible makes right later shift when
occludes in centric whats the cause:

-Mesial cusps of upper teeth

-Distal cusps of lower

-Palatal surface of upper buccal cusps **

-Buccal surface of upper platal cusps

15. probe around healthy implant without bleeding:

-3 **
-5

-7

16. pt on hypertensive drugs for many years whats the likely complication
that will affect the planning for complete denture :

-Dry mouth **

-Gagging

***17. A 20 year old pt with class 3 and 6 mm overjet and anterior cross
bite whats the ttt:

-Orthographic surgery

-Camouflage

-Headgear
18. most important xray for pts with -facial asymmetry :

-Panoramic

-Ceph

-Anterior posterior ***

Note: Cone beam computed tomogram imaging offers more precise


information regarding the features of mandibular asymmetry than
conventional PA cephalograms. Therefore, when a visible chin deviation is
present, CBCT scan should be preferred.

19. pt came next day after you did composite for him, you did liner with
caoh and base and then compsite, both the restoration and too are intact
but the pt feels sever pain and very tender to biting and percussion Whats
the cause :

-Polymerization shrinkage

-Irreversible pulpitis because of the liner

-Hyper occlusion***

20. question about the definition of enameloplasty

21. a pic of thro and thro defect the brush enters the furcation easily and
ask about what was the procedure:

-Tunneling ***

-Furcation plasty
** 22. pt came wants to replace upper lateral and 11 is proclined and endo
treated, 11 and 13 have proximal and lingual caries whats the best ttt :

-Single implant

-3 units fpd from 11 to 13 **

- extract 11 and bridge

* similar question

patient with missing lateral and misalignment in central and proximal and
facial caries in both canine and central what you should use?

A-single implant

b-fixed denture

c-resin bonded fixed denture

d-removable

‫😅😅الشبيه‬

23. whats of the following is yet to be confirmed as a putative risk indicator


of clinical attachment loss :

- bleeding on probing **

- presence of abundant plaque

- malocclusion

- clinical attachment loss


24. most common interference while crown lengthening distal to lower
second molar?

Mylohyoid ridge

Extrnal oblique ridge

Lingual nerve**

***25. pt with canine space infection suddenly had proptis of eyes and
paralysis

Cavernous sinus thrombosis **

Periorbital cellulitis

Orbital cellulitis

Acute exacerbation of the canine space infection **


‫ ﻣﺎرس‬١٩ ‫ﻣﻨﺎﻗﺸﺔ‬

1. - the dentist took alginate impression and disinfect it for 10 mins then left it
without cover for one hour what would happen to the cast after pouring?
- brittle porous cast
- the cast will be larger than pt mouth

✅✅
- the cast will be equal to pt m
- the cast will be smaller

2. A 20 yr old female with open bite dentist told her the ttt will take 18 months but
she wants less time, what can be done to accelerate the ortho ttt :
-Crown lengthening

✅✅
-Occ adjustment
-Bone decortication

3. Chemotherapy, how many days b4 chemotherapy we can do extraction?

✅✅
**You usually start radiotherapy 4 to v6 weeks after surgery. If you are also
having chemotherapy, radiotherapy is given after chemotherapy .
••

✅✅
Some important notes about Ideal time for extraction in pt.:
Taking radiotherapy: Before 2week and ideal time 21 days.

Taking chemotherapy: Before 5days in maxilla and 7days in mandible. ✅✅✅

4. medication we don’t give to pregnant woman?

🛑
Drug with pregnant:
*Contraindicated*
Asprin with high dose
Ibuprofen
Diclophenac

🛑*Indicated*
Acetaminophen is usually use
Asprin with low dose

5. Some doctors take the certificate without attending the course or lecture, this

✅✅
violation of what ???
Vercaity

6. If the patient during examination say (( aaaaah )) and the half of the soft palate
move and the other side not moved and the uvula more toward the right side ,

✅✅
Which nerve is affected !
Vagus
, facial ,
glossopharyngeal .

7. Best way to remove gutta percha ?


h file,
k file,
reamer ,
heat

**if coronally reamer


apically H file
8. Bur used for finished in proximal surface?

✅✅
Rubber cup
Disc
-
-

9. different between cemented implant and serrated implant?


‫وﺟﺎﻧﻲ ﺳﺆال ﺻﺮاﺣﻪ ﻣﺎﺗﺬﻛﺮ ﺑﺎﻟﻀﺒﻂ ﺑﺲ ﻣﻤﻜﻦ ﺣﺪ ﺟﺎه وﯾﺘﺬﻛﺮ اﻟﺒﺎﻗﻲ ‪ /‬ﺷﺨﺺ ﺗﻌﺮض ل ﺿﺮﺑﻪ ) ﻟﻜﻤﻪ ﻓﻲ ‪10.‬‬
‫اﻟﻮﺟﻪ ( وﺗﺤﺮﻛﺖ اﻟﻤﺎﻧﺪﺑﻞ اﯾﺶ ﺻﺎر ؟‬
‫‪-articulatr disc slop‬‬
✅✅
-TMJ movement
-TMJ deviation

****—In addition to questions about these topics—

*time of neonatal teeth present


* Morphodifferentiation
* Class I malocclusion
* Normal occlusion
* Implantsssssss and the spaces
* Akinosy technique
* 2-3 questions about gingivectomy and crown lengthening and extrusion
* Sinus, oroantral communication

*** question about Phenotype

✅✅
This term comprises two terms, gingival phenotype (gingival thickness and
keratinized tissue width)
*Wilson and S curve in prostho?
*Non working condyle position
*apexification case with radiographs
> histopathology of pemphigus vulgaris
Pemphigoid
- while inserting the denture pts mandible makes right later shift when occludes in
centric whats the cause:
-Mesial cusps of upper teeth
-Distal cusps of lower

✅✅
-Platal surface of upper buccal cusps
-Buccal surface of upper platal cusps

➡pic of gingival recession


Not extending to mgj(3mm)and has moderate bone resorption?
➡ha1c of moderate controlled diabetes:

✅✅
A.5-6%
B.7-8%.
C.8-9%
➡‫ ﺷﻬﻮر اﯾﺶ اﻟﺴﺒﺐ‬٤ ‫ﺑﻌﺪ‬1.5 ‫اﻣﺒﻼﻧﺖ واﺣﺪ ﻓﯿﻬﻢ اﻟﺒﻮﻛﯿﺖ ﻛﺎن‬2 ‫وﺣﺎﻟﻪ ﻋﺎﻣﻠﻪ‬

A 54 - year old healthy man had 2 implants placed in the lower right quadrant 4
months ago. During the second stage procedure, the distal implant displayed 4-5
mm crestal bone loss and exhibited significant mobility. The
mesial implant was within normal limits Which of the following is the most likely
reason for such bone loss?

✅✅
A. allergic reaction
B. excessive surgical trauma
C. the patient's medical condition
D. healing abutment was not
installed
‫🏼☝🏼‬
‫☝ﺣﺴﯿﺖ اﻟﺴﺆال اﻻول ﺑﺸﺒﻪ ده اﻟﺴﺆال‬

‫✅✅‬
‫ﺑﻌﻤﻠﻬﺎ ﻓﯿﻦ اﻧﺘﯿﺮﯾﻮروﻻ ﺑﺮﯾﻤﻮﻻر وﻻ ‪ imediet implant‬وﻟﻮ ﻫﻌﻤﻞ➡‬
‫‪Anterior‬‬
‫اﺟﺎ ﺳﺆال ﻋﻦ اﻟﻄﻮل اﻟﻌﺎﻣﻞ ﻟﻨﺎب ‪٢٢‬ﻣﻞ ﻛﻢ ﻻزم➡‬

‫**اﻟﺴﻮال ﻧﺎﻗﺺ ﺑﺴﺎل ﻋﻦ ﻃﻮل اﻟﺒﻮﺳﺖ ﻛﻢ اذا ﻛﺎن ﻃﻮل اﻟﻜﻨﺎل ‪wl 22‬‬

‫ﻧﻘﺴﻢ ‪ ٢٢‬ع ﺗﻼﺗﺔ وﻧﻀﺮب اﻟﻨﺎﺗﺞ ﻓﻲ ‪١٤.٦ =٢‬‬

‫‪**Post= tow third of canal‬‬

‫🛑🛑🛑🛑🛑🛑🛑🛑🛑🛑🛑🛑‬
‫⛔⛔اﺳﺌﻠﺔ ﻣﻜﻜﺮة⛔⛔‬
‫👇🏾👇🏾👇🏾👇🏾‬
‫اﺟﺎ ﺳﺆال ﻋﻦ اﻟﻄﻮل اﻟﻌﺎﻣﻞ ﻟﻨﺎب ‪٢٢‬ﻣﻞ ﻛﻢ ﻻزم ﻧﺸﯿﻞ ﻛﻮﺗﺎ ﻋﺸﺎن ﻓﺎﯾﺒﺮ ﺑﻮﺳﺖ‬
‫ﺳﺆال ﺣﺸﻮة اﻣﻠﻐﻢ ﻧﺺ ﻣﯿﻠﻲ اي ﻧﺤﻂ ﻻﯾﻨﺮ ﺗﺤﺘﻬﺎ ‪..‬‬

‫ﺳﻮال ﻋﻦ ﺣﺪﺑﺎت اﻟﺪﻋﻢ ‪..‬‬

‫ﺳﺆال ﻋﻦ ﻛﻢ ﻻزم ﻧﺸﯿﻞ ﻣﻦ اﻟﺤﺪﺑﺎت اﻟﻤﻮﻻر ﻋﺸﺎن ﺗﺎج ﺧﺰف ﻣﻌﺪن ‪..‬‬
‫ﺳﺆال ﻋﻦ ﺣﺪﺑﺔ ﻛﺮﻟﺒﻲ و ﺗﻮﺿﻌﻬﺎ‪..‬‬

‫ﺳﺆال ﻋﻦ ﺗﻌﺮﯾﻒ اﻟﻨﻤﻮذج اﻻول ﻟﻠﻄﺒﺎق ﻋﻼﻗﺔ اﻟﺮﺣﻰ اﻟﻌﻠﻮﯾﺔ ﺑﺎﻟﺴﻔﻠﯿﺔ‬


‫‪ AH PLUS‬اﯾﺶ اﺳﺎس‬

‫اﯾﺶ ﻧﺴﺘﺨﺪم ﻋﺸﺎن ﻧﻌﺮف اﻟﺨﺮاج ﻣﻦ اي ﺳﻦ ﻛﻮﺗﺎ او ﻓﺎﯾﻞ او ﺑﯿﺒﺮ ﺑﻮﯾﻨﺖ‬

‫ﺳﺆال ﻋﻦ ﻏﯿﻠﺪن ﻏﯿﺘﺲ ﻣﻘﺎس ‪٣‬‬


‫ﺳﺆال ﻋﻦ ﺷﻜﻞ اﻟﺤﺠﺮة اﻟﻠﺒﯿﺔ ﻟﻀﺎﺣﻚ ﻓﯿﻪ ‪ ٣‬اﻗﻨﯿﺔ‬

‫ﺳﺆال ﻋﻦ ﺷﻮ ﺑﻨﺴﺘﺨﺪم ارﯾﻐﯿﺸﻦ ﻻﺑﻜﺲ ﻣﻔﻮح‬


‫ﺳﺆال ﻋﻦ ﻣﺴﺎوء اﻟﻜﻮﺗﺎﺑﺮﻛﺎ‬

‫ﺳﺆال ﻋﻦ ﻣﺮﯾﺾ ﺳﻮا ﺣﺸﻮة و اﺗﺼﻞ او ﺟﺎﻋﻨﻚ ﺑﻌﺪ ﻛﻢ‬


‫ﯾﻮم وﻗﻠﻚ اﻟﺤﺸﻮة ﻟﻮﻧﻬﺎ ﻏﺎﻣﻖ ﻟﯿﯿﺶ ؟‬
‫ﻃﺮﯾﻘﺔ اﻟﺘﻌﺎﻣﻞ ﻣﻊ ﺑﯿﺸﻨﺖ ﺗﻘﯿﺄ ﻋﻠﻰ اﻟﻜﺮﺳﻲ و ﺳﺆال ﺛﺎﻧﻲ ﺗﻘﯿﺄ ﻋﻠﻰ اﻻرض‬

‫وﺳﺆال ﻋﻦ ﻣﻤﺮﺿﺔ اﻧﺠﺮﺣﺖ ﺑﺲ ﻣﻮ ﻣﺘﺬﻛﺮ اذا ﻛﺎن ﻓﻲ ﻋﺪوى ﺑﺎﻟﺠﺮح وﻣﻜﺘﻮب اﻧﻮ اﻟﻤﻤﺮﺿﺔ ﻣﺎ ﻋﺎدت اﺟﺖ‬
‫ﻋﺎﻟﻌﯿﺎدة ﺑﺎﻟﺨﯿﺎرات ﻣﻜﺘﻮب اﻧﻮ ﺗﻌﻄﯿﻬﺎ ﻋﻄﻠﺔ او ﺗﺨﺒﺮ اﻟﻤﺪﯾﺮ او ﺗﻜﺘﺐ ﺗﻘﺮﯾﺮ ﯾﻌﻨﻲ ﻣﺎﻛﺘﯿﺮ ﻣﺘﺬﻛﺮ‬

‫وﺳﻮال ﻋﻦ اﺳﻮء وﺳﻂ ﻟﺤﻔﻆ ﺳﻦ ﻣﺨﻠﻮع‬

‫اﻻﺟﻮﺑﺔ‬‫🛑🛑‬
‫‪ _1‬ﻃﻮل اﻟﻔﺎﯾﺒﺮ ﺑﻮﺳﺖ ‪ 2/3‬ﻣﻦ ﻃﻮل اﻟﻜﻨﺎل = ‪14.3‬‬
‫‪ -2‬ﺗﺤﺖ اﻻﻣﻠﻘﻢ ﻧﺴﺘﺨﺪم ‪) caoh‬ﺗﺎﻛﺪو ﻣﻨﻬﺎ(‬
‫‪mandibular buccal... Maxillary palatal -3‬‬
‫‪functional cusp 2m... Non functional 1.5mm - 4‬‬
‫‪Class‬‬
‫‪(cusp of carbily (uper 6 palatal serfice -5‬‬
‫‪(class 1 (MB Cusp of uper 6 occlod in buccal groove of lower 6 -6‬‬
‫‪AH plus is epoxy resing-7‬‬
‫‪gutta perch - 8‬‬
‫‪gg 3 saize 0.90 - 9‬‬
‫‪traingular -10‬‬
‫‪CHX -11‬‬
‫‪Shrinking -12‬‬
‫‪ -13‬اﺧﺪﻧﺎ اﻟﻠﻮن ﺑﻌﺪ رﻛﺒﻨﺎ اﻟﺮﺑﺮ دام‬
‫‪Flow manofactory instraction.... Intermediant -14‬‬
‫‪✳✳ - 15‬‬
‫‪ - 16‬اﺳﻮاء ﺷﻲ ‪ water‬وأﻓﻀﻞ ﺷﻲ ‪hpss‬‬

‫‪Dr.Afnan‬‬ ‫💕‬
‫اذﻛﺮوﻧﻲ ﺑﺪﻋﻮة‬
‫مناقشة ‪ ٢٠‬مارس‬

‫ز‬
‫كومتت استخداما️▶‬ ‫ر‬
‫ًاكت نوع‬

‫*** ‪Hybrid‬‬

‫‪Flow‬‬

‫‪Micro‬‬

‫‪Nano‬‬

‫ز‬
‫‪rbd‬ف ال ‪ complcation‬ر‬
‫اكت حاجة بتعمل️▶‬ ‫ي‬
A. Major Connector

B. Clasp ***

C. Retainer
‫ما ر ز‬
D. ‫متكر‬

Note: RPDs suffered more deterioration than the abutment teeth


of the clasp retained RPDs. caused these complications. and tooth
mobility. Plaque, Gingivitis, Calculus, Tooth mobility, Probing
Depths and Gingival recession between abutment and non-
abutment teeth

▶️‫فيه سؤال جاء‬.

‫ وجاء عنده بالك ويحتاج‬.‫ مريض كىل كمل زراعة من شهرين‬Scaling & RP ‫وعنده‬
‫ سيمبل‬Caries‫؟‬

- ‫ شهور وبعدها تعالجه‬6 ‫*** تنتظر‬

- ‫تعالجهم كلهم عادي‬

- ‫ تعمل‬scaling ‫وتأخر التسوسات‬

- ‫ تعمل التسوسات وتأخر‬scaling

Note: • Defer dental treatment: After transplantation, routine


dental treatment should be postponed until maintenance dose of
immunosuppressive agents is reached

Emergency treatment only during first 6 months


▶️Muscles responsible for Border molding in lingual area

▶️Radiograph for #44 #45 with open apex endo test necrosis?
‫ حطيت‬root Regeneration

Apexofication + pulpectomy✅✅✅
▶️Office bleaching works on?

Enamel ***

Dentin

Both enamel and dentin

Note: Dentin can NOT be whitened


▶️Most part of the enamel

Enamel Rod ***

Enamel tuft

Interrod Enamel
…..
▶️ A 3-year-old child ingested a toothpaste tube. What’s the
management?

Drink milk or induced vomiting I don’t remember the other


choices

Drink milk✅✅✅if part of toothpaste


Take to immediately hospital >> if whole tube

▶️ Child is caries free and has good oral hygiene maintenance.


What is the fluoride recommendation for this patient?

a) Brush once with fluoridated toothpaste

b) Brush twice with fluoridated toothpaste ***

c) Brush twice with fluoridated toothpaste and fluoride


mouthwash

D) Brush once with fluoridated toothpaste and fluoride


mouthwash

▶️‫صورة قريبه لدي‬


▶️automated way for measurement of sulcus or pocket: ▶️Florida
▶️qualitative light induced fluorescence uses

A) Initial caries

B) Enamel and dentin

C) Incipient cares***
▶️Hypothyroidism patient, not taking medication, so what would
happen ▶️ myxedema coma

Note: ✅✅✅✅✅✅

Myxedema also refers to the swelling of the skin and soft tissue
that occurs in patients who are hypothyroid. Myxedema
coma occurs when the body's compensatory responses
to hypothyroidism are overwhelmed by a precipitating factor such
as infection.
▶️when to use cavity test? The last option was if all the other
investigations don’t work
‫ر‬
‫الباف‬
‫ما اذكر ي‬
Note: The test cavity technique is only used as a last resort when
results produced by all other methods above are inconclusive.
High speed burs are used without anaesthetic, drilling through
enamel or restorations to dentine

▶️ Bridge with 0.3 mm gap in the margins. what to do?

we remake

Seal with cement ***


‫ر‬
‫الباف‬
‫ما اذكر ي‬

▶️which one of the following have the highest pH ?

- normal saline
- sodium hypochlorite
- EDTA
- Calcium hydroxide*****
- Phosphoric acid

Note: **Ph:

Sodium hypochlorite- 11 to 12

CHX - between 5.5 and 7


Ledermix- 8.13

Formocresol - 12.45

Eugenol - 4.34

AH plus - 7.81 to 7.17 at 3 hours to 24

Normal tissue - 7.4 in presence of inflammation decrease to 4.4 to


5.5

Critical ph for demineralization- 5.5

Normal saline =5.5

Sodium hypochlorite =11

EDTA =4 to 6

Calcium hydroxide= 12.8

Phosphoric acid = 2.4

▶️19. According to ante’s law which one is the best when you have
#4 and #5 missing. What would be the choice for bridge?

A) #3,4,5,6 ***

B) #3,4,5,6,7

.....

.....

Note: ‫ الجواب‬5 & 4 ‫ اذا کان المفقود‬A


‫‪ B‬واذاکان المفقود ‪ 6 & 5‬الجواب‬

‫بيشنت مركب كل فمه كراون فوق وتحت وبتسوي له جديد وش تستخدم ️▶ ****‬
‫اي نوع‬

‫واالمالقم وش كان السبب ؟ الخيارات كلها كانت ‪pin‬بيشنت انكرس معاه ال️▶ ****‬
‫تتكلم عن مكان ز‬
‫الب‬
▶️Master cone number 20 diameter at the tip

- 0.02

- 0.2 ***

-2

- 20

▶️Cases indicated to wear a mask …

Hepatitis C virus

Tb ****

.....

....

▶️ Adhesive in CD patient used for what type of palate?

A) u-shaped palate

B) V-shaped palate

C) Cleft palate

D) Flat palate ****

Note: D

‫ الن هو اقل نوع‬retention


‫ مارس‬21 ‫مناقشه‬

▶️pt with complete denture and the palatal is effect ‫يعني البلت متاثر جدا ايش‬
‫السبب‬

A denture unfit ***

B allergic from acrylic

C monomer of acrylic.....?

D?

▶️how to make the rest more flexible?


Increase length ***

Increase diameter

question about alginate, monomer to polymer ratio 1:1 what does that ▶️
cause? Shrinkage
▶️

allergy

With out x ray radiopaque in middle of the tooth to the root What's ▶️
this and causing

A which is the problem may be occurring

B. The canal is meeting together

C. The canal is calcified


▶️

‫ او ايش المشاكل اللي راح تقابلك‬D

‫طبعا هذا سوالين مخلوط بسوا‬


pic thick white lesion on the palate▶️

hairy leukoplakia

Pemphigus vulgaris

looks like candida ▶️

Hairy leukoplakia in tongue not palate -

pic upper and lower inflamed, erythematous gingiva with bleeding but ▶️
?no bone resorption, how to treat this case

Systemic corticosteroids

topical corticosteroids
▶️
amalgam restoration class v deep, for 7 months, first 6 months it was ▶️
ok but started to be painful in the last month. What is the reason?
▶️Shrinkage / pulp exposure

patient came complaining of a fractured palatal cusp in upper 6. How ▶️


?do you restore it

Composite

inlay

onlay
▶️

?patient fell on his chin. What is most likely to fracture ▶️


▶️

▶️Condyle

▶️Symphysial area
...

...
deep cavity and you want to restore with composite. What is the ▶️
?? technique or layer thickness

2 3mm-✅✅

12mm-

‫لو قصده علي‬,✳✳

Thickness of layer of composite

‫ مم‬2 ‫ ل‬1 ‫اعتقد انها من‬

Scenario about in xray present mesial root abscess in 3rd molar and ▶️
the case is hopless and need to extract But the patient said the last
‫ اخر مره لمن خلع نزف كثير‬.time he extract ed he got a lot of bleeding

A) ‫كتب ادويه كثير بالخيارات‬

)Bone of the options the hemostat material and suture the socket✅✅
‫️▶المريض ماعجبه كالمك وقرارك فيطلب االشعه‬

‫تعطي كل نسخ االسعه‬A✅✅

‫ تعطي اصول االشعه‬.B

‫ تعطي نص النسخ‬.C

hypertensive patient, have diabetes and other things )I don’t ▶️


remember) and he’s yellowish and swollen fingers )feels like a heart
disease). What is you diagnosis? Heart disease / liver disease✅ ✅

▶️Attration in pt with class 3

‫ كالس بسبب االسنان في اترشن معه شخص‬3 ‫حشوه عامل وهو غامقه صارت واالسنان‬
composite... ‫له تعملي ايش‬

A Veneers

B. Composite✅

‫ال في فالخيارات‬crown ‫هيكون االصح‬

Pt with pigments in all teeth with spacing between the teeth ▶️


A. Crwons with preparation✅

B. Crwons without preparation

✳ mother brought her daughter who fell and fractured her anterior ▶️
?teeth. You suspect that she has hepatitis. How do u manage

Give analgesic and refer pandemic center


Take full precautions and treat her✳

Give analgesic and Send for bleed test

Tooth fractures in enamel and dentine andYou did her the necessary ▶️
.treatment

?when the next date for the follw up


tooth gray and canal obliterated▶️

Amelogesis imperfecta


Dentinogenisis imperfecta✅
minimum time to sterilize surgical intrument in 131▶️
The minimum exposure time is required for sterilizing instruments is
.minutes at 131ºC or 60 minutes at 136ºC 50

? Why the older age have widening in the pdl▶️

Note: The normal width of the PDL ranges from 0.15 mm to 0.21 mm,
which may decrease with age. 1,4 Widening of the PDL is one of the most
important changes in the circumdental structures and may be related to
different abnormalies
▶️‫ عندو بيشنت‬missing ١٤ ‫ و‬١٦ ‫؟ يسوي ايش‬

fixed from #13 to #17✅✅

fixed from #13 to #15 with cantilever

▶ a 5 year old patient is class iii. How to treat?

Treat the patient

Wait✅✅

▶️ ‫ ال‬pin With amalgam restoration


✳ ▶️2 crowns with different cervical line

which one is better


‫⬆⬆ هذا‬

far from ‫ ال‬cementume area

And preserve biological width


▶️first sign of chemical burn in gingiva?

A. White wrinkle ✅✅

B. epithelial desquamation

C. ...,. Fibrin

Note: The typical chemical burn is manifested as superficial white to yellow,


wrinkled lesion. Desquamation of the underlying tissue due to necrosis
depends on the duration of exposure to chemicals. As the duration of
exposure increases, the necrosis of the tissue increases

▶️cd pt with soft tissue trauma time for treatment

Aftr24 hrs

Give tissue conditioner and final impression on next appointment ✅✅

Aftr 1 week

▶️*rpd component with matrix and patrix

Connector (qn not fpd)

Attachment✅✅
‫ مارس‬٢٢ ‫مناقشة‬

✅✅
1- Complete epithelialization occurs after 7-14 days,

2- Optimum crown root ratio 2:3 ✅


3- Pt MI 9 months ago and on warfarin , he wanted esxtraction?
What do you do?

✅✅
-reffer for physition
-you can extracted if INR 2-3

4- Rct with spep back tequnic


Master con #55,
Master file ?!

✅✅
-50
-55

**Master cone ‫بيكون نفس‬


‫ مقاس‬MAF = master apical file
‫‪ emergency pregnant woman‬سؤالين بيتشابهو عن ‪5-‬‬
‫وحدة لها ‪ ٦٨‬يوم ووحدة في الشهر الخامس؟‬

‫نأخر الشغل لبعد الوالدة‪-‬‬

‫✅✅‬
‫نديها ايبوبروفين‪-‬‬
‫‪ rct‬نبدأ لها عالج‪-‬‬

‫✅✅‬
‫‪6- Burshing technique with recession: modifide stilmans‬‬

‫✅✅‬
‫واضحة ‪ x-ray‬صور ‪mishap‬كم سؤال عن ال‬
‫‪Ledge‬‬
‫‪Furcal perforation‬‬
✅✅
7- determine molar relationship in primary dentition?
Primary second molar
8- Angle of occlusion plane orientation :


For incisal .. zero
For lateral condylar .. 15 - 20
For protrusion .. 30-40

✅✅
9- geriatric pt. CD which teeth?
-0
-25
-30
33

**Non anatomical teeth are preferred

✳️✳️10- ‫سيناريو على االمبالنت‬


‫ انه بيشنت عمل امبالنت وكانت ال‬torque=35NCM
‫ وفي ال‬maximum inter cuspation ‫ عنده‬slight contact ‫ وكمان بال‬eccentric movement ‫عنده‬
‫كونتاكت‬
‫ رجع بعد فترك وعنده‬loose ‫ايش السبب؟‬

☝🏼☝🏼☝🏼☝🏼
‫ او االكسنترك موفمنت‬..‫ او الكونتاكت بالماكسمم انتركسبيشن‬..‫بسبب التورك‬

✅✅‫االجابة التالتة صحيحة‬


‫التورك مقبول و اكتر شي اربعين بعدها ممكن ينكسر السكرو‬
..‫التورك مقبول و اكتر شي اربعين بعدها ممكن ينكسر السكرو‬
Loos ‫بكون على مرحلتين‬

external forces such as mastication applied to the screw joint causes slippage,
contributing to the release of preload of the screw

second stage of loosening involves continual preload reduction below critical


level, allowing threads to turn and the loss of intended screw joint junction

▶️‫الحركات الالمركزية بسموها اكسنتريك و هية حركات المضغ و تقديم الفك لالمام‬
✅✅✅✅✅
Protrusion
11- Preparation of pin hole?
Prepindcular to palpal floor
Parallel to long axis of adjecent tooth

‫وفي خيارين كمان مامتزكراهم‬

12-Position to operate in the upper jaw?(highlighted areas in the pic)


8-9 oclock
12 oclock
1 oclock
13-Which of the following areas are not self-cleansing on a tooth Crown?
Inclined planes of cusps and Ridges
Marginal Ridges

✅✅
Fossae
Pits and Fissures

✅✅
14- Which type of bone around apical third of root?
Cortical
Periostial
15- Tooth discoloration due to trauma
How do bleaching?

✅✅
Vital
Non vital
Microabrasion
✅✅
16- small fructure cusp?
Just do smoothing
Full crown
Not do tt

✅✅
17- Bur used to occlusal groove for crown?
Round end tapperd diamond
Fissure
Inverted cone

18-
24.Dentist ask assistance to clean water line in clinic to prevent future

✅✅
contamination, what is the importance part to clean:
a. Stagnant areas
B. Outgoing waterline
C. Ingoing waterline
D. None of well sealed waterlines

✅✅question about bennett movement


✅✅questions about pontics

19- 4 years old pt came with father complain of gray teeth (central incisor) after
trauma ,not response with cold , X-ray show normal apical tissue .
What best tx?
• Extraction
• Pulpectomy

✅✅
• Pulpotomy
• Leave it until show change or abnormal in x-ray

✅✅
20- When check occlusal FPD
Before cementation
Before check of countor
21- ‫جاب لي سيناريو مشكلة في ال‬tonge ‫ وجاب معاها‬labrotary test

✅✅
‫ وعندو‬low MCV ‫ايش حتعطيه عالج؟‬
Iron
Vit B
Folic acid

✅✅
22-resting tremors and impaired posture:
Parkinson
Alzheimer's
Multiple secleroesis

✅✅
23-Behçet’s syndrome?
= orogenital ulcerations , Eye lesions , skin lesion , Positive pathergy test

✅✅
24-‫ سيناريو بيشنت‬pedo ‫ وعايزين نديه‬anesthesia‫؟‬
Lidocien
Articain
Mupvicain

✅✅
25-Shade selection?
Florecent

26-Carbohydrate in moth

✅✅
Duration
Frecaunce
Form

27-Disinfict GP ?
Naocl
Gltraalfahide
‫مكتوب نسب بس مامتذكرة‬

**desifection of impression =
Alginate + compound + polyether = 0.5%

NaOCl 10min ZOE + polysulphide + PVS =2% Gultaraaldehyde

28-TB in air
-Up to minutes

✅✅
-Up to 2 hours
-Up to 4 hours *

29-Old patient with palatal hyperplasia the patient is disabled can’t move his
hands, reason for the hyperplasia:

✅✅
-Cant clean the denture
-wearing it for long time and unclean
30-Base under composite for deep class5?,

✅✅
Zinc oxide
Glass ionomer
Caoh

31-Matrix band should be over margin by


0.5 mm

✅✅
1mm
2mm

32-Which band best for composite class 5?


Mylar
Tofflemier with ultra thin band

✅✅
Contoured matrix band
No need

✅✅
33-Autoclave is?
a) Pressed steam
b) Pressurized vapor

34-Question about histology of lichen planus


✳️✳️
‫صورة اشعة فيها اسنان‬
erupted and non erupted teeth
And radiolucens

✳️Cleidocranial dysplasia?
✅✅Herpes simplix viruse
✅✅
‫االرقام‬
Stanuce flouride
Hydrogen peroxide
Sodium carbamid

Dr.Afnan 💕
‫اذكروني بدعوة‬
‫ مارس‬٢٤ ‫مناقشة‬

🛑burning of tounge> exflation cytology✅✅


🛑anti ssa >
sjogern✅✅
lupus or
⁃ Anti ssa? Sjogren
DescriptionAnti-SSA autoantibodies are a type of anti-nuclear autoantibodies that
are associated with many autoimmune diseases, such as systemic lupus
erythematosus, SS/SLE overlap syndrome, subacute cutaneous lupus
erythematosus, neonatal lupus and primary biliary cirrhosis. They are often
present in Sjögren's syndrome
🛑defntion of autonmy and patrnalism and apexgensis

🛑perio classificaton✅
🛑concousion>
come with mobilty

✅✅
or
no
🛑class 2 age 11 retroded mandible> functioal✅✅
or cervical headgar
🛑leat esthatic in pedo> compsite
or compmer
or gic
or silver amalgam ✅✅
🛑amalamg extracyion> biohazerd✅✅
🛑pic of fordycaz
Fordyce spots (also termed Fordyce granules) are visible sebaceous glands that

✅✅
are present in most individuals. They appear on the genitals and/or on the face
and in the mouth

🛑violation of bologic width.


Biologic width violations can be corrected by either surgically removing bone

✅✅
away from proximity to the restoration margin, or orthodontically extruding the
tooth and thus moving the margin away from the bone

**biological width =2.04


epithelial attachment =.97
connective ts attachment =1.07

🛑TP in clinic > 4or 6 hour✅✅


🛑recurent of puogenic granuloma > 15%✅✅
🛑pinc discolartion> internal resorbtion✅✅
🛑TFO secondry > 30-50✅✅
**Secondary occlusal trauma ‫ بتعمل‬bone loss 30-50 %
**Primary truma from occlusion =resultsfrom alteration in occlusal force high
filling

Secondary truma from occlusion = reduce ability of tissue to resist force

Primary trauma from occlusion: tissue reaction around a tooth with normal
periodontium
height

- Secondary trauma from occlusion : tissue reaction around a tooth with reduced
periodontium
heigh

30-50% bone loss = secondary trauma

🛑zinc posphate>
sensitive✅✅
pulp necrosi or
🛑avoid in temroray crown ➡️ Polymethyle methacrylate✅✅
🛑12 flutid bur>
macro abraion✅✅
micro or
🛑implant to natural teeth destancs.... 1.5 mm✅✅
🛑D16 of file 10
0.22✅✅
..
..
D0= 0.10
‫ و‬d6 = 6 × 0.02 = 0.12
‫نجمعهم‬
= 0.22
‫? ‪**What is the D 15 for file 25‬‬

‫المعادلة‬

‫‪ :‬الزم يكون عندك اثنين من المعطيات‬

‫)‪ -١‬رقم الفايل ‪ ،‬مثال ( ‪10,25,50,70‬‬

‫)‪ ( D8 , D15 , D4‬كم ‪ ،‬مثال ‪ -٢ D‬رقم ال‬

‫اول شي ناخذ رقم الفايل ونحوله مثال فايل‬


‫رقم ‪0,25 = 25/100 <<<< 25‬‬

‫‪ D15‬مثال ‪ D ،‬بعده نحول ال‬

‫‪ d‬نضرب الرقم الثابت الي هو ‪ 0.02‬ب رقم ال‬

‫‪0.02 x 15 = 0.30‬‬

‫‪ d‬بعدها نجمع الفايل الي طلعناه مع ال‬

‫‪0.25+0.30 = 0.55‬‬

‫‪🛑voxel size at least‬‬


‫‪👉🏻absolute minimum Voxel size for endodontic should be *0.2mm*.‬‬
‫‪👉🏻maximum Voxel size for endodontic should be *0.4mm*.‬‬
‫)‪different voxel resolutions (0.4, 0.3, and 0.2 mm‬‬

‫حكيم ال ‪ vexel‬هو عبارة عن قياس حجم يستخدم بالتصوير ثالثي األبعاد‬

‫‪? What is the D 15 for file 25‬‬


‫المعادلة‬

‫الزم يكون عندك اثنين من المعطيات ‪:‬‬

‫‪ -١‬رقم الفايل ‪ ،‬مثال ( ‪)10,25,50,70‬‬

‫‪ -٢‬رقم ال ‪ D‬كم ‪ ،‬مثال ( ‪)D8 , D15 , D4‬‬

‫اول شي ناخذ رقم الفايل ونحوله مثال فايل‬


‫رقم ‪0,25 = 25/100 <<<< 25‬‬

‫بعده نحول ال ‪ ، D‬مثال ‪D15‬‬

‫نضرب الرقم الثابت الي هو ‪ 0.02‬ب رقم ال ‪d‬‬

‫‪x 15 = 0.30 0.02‬‬

‫بعدها نجمع الفايل الي طلعناه مع ال ‪d‬‬

‫‪0.55 = 0.25+0.30‬‬
🛑crown lenthining of 7 > injury lingaul nerve✅✅
🛑Proper documintation should be:
Subjective

✅✅
Objective
Complete

🛑
‫تاثير التروما عالبريو‬

🛑white steria... Lichen planus ✅✅


🛑best impressions to onlay?

🛑liver disease with test and no pt in choise


‫اذا البيشنت بياخد‬

Aspirin > Bleeding time( BT )

Warfarin > INR + PT

Renal dieseas, heparin > PTT


Liver disease > BT + PT✅✅

🛑slop technic... Same lingual opposite buccul


‫مشان تشوف القنوات وفي حاله الكيرف‬ ✅

🛑134 autoclave in minutes... 3 min✅

🛑chemotherapeutic > mucositis✅✅


🛑Down syndrome take consent from caregiver✅✅
🛑cell of pdl in resorption and deposition > fibroblast✅✅

🛑magnitude > elliptical✅✅


‫🛑‬
‫✅✅‬
‫سوال عن البورد مولدينح؟‬
‫‪Green stick compound‬‬
🛑‫وكمان الحركه ب‬non warking sid
‫والحركه ب‬working side

➡️ ✅
➡️ ✅
Working laterally
Non working medily
‫‪🛑Complication if single crown only..‬‬
‫✅ ‪Un aesthetic‬‬

‫?‪🛑Complication if lip is long‬‬


‫✅✅‪F and v sound‬‬

‫جاب مراه خامل بشهر االول ومعها الم حاد ايش تعطها مسكن وجلها للوالده او تعطها مضاد تحولها ومذكرش‬ ‫🛑‬
‫✅✅‬
‫الباقي‬
‫تعمل ‪ pulptomy‬وتكتب لها ‪parathetamol‬‬

‫جاب مريض ياخذ اسبيرين ‪325‬ايش تشتغله‬ ‫🛑‬


‫🛑‬
‫وجاب سوال نخظ باالول جلفس وبعدين جاون المهم مرتيات عكس الجاون‬
‫✅✅‬
‫جاون االول واخر حاجه نلبسها الجلفز عشان متلمسش غير العيال‬
‫ولما نخلص اول حاجه نقلعها الجلفز‬

‫🛑‬
‫اذا ركبنا كراون لسن واخد فقط هل الي نخاف منه اللون او االنالميشن او حدوث العصب‬

‫اللون✅✅‬
‫جاب متى مانستخدم ‪ anterior bit plane‬هل في ‪ open bit‬او‪deep bit‬او‪cross bit‬او‪normal‬‬ ‫🛑‬
‫‪Deep bite‬‬‫✅✅‬
‫جاب ايش اسم المسل الي تخرك الفل يمين ويسار‬ ‫🛑‬
✅Lateral pterygoid
‫ايش سبب لنحوال نيرف او تيريحويد‬trusmus ‫جاب عمل تخدير دكتور الكثر من سن فجاه المريض زاد لديه‬ 🛑
‫نيرف او تيرفات ثايته‬

Trismus after anaesthesia is usually caused by intramuscular injection of the


anaesthetics in the pterygomandibular space. It can occur even 2 – 5 days after

✅✅
inferior alveolar block anaesthesia. Affected muscles are usually either the lateral
pterygoid muscle or the temporal muscle.
🛑herpitic gingivistomatitues
‫‪Dr.Afnan‬‬ ‫💕‬
‫اذكروني بدعوة‬
‫ مارس‬٢٥ ‫مناقشة‬

1- The question of rpd pt who had 2 previous dentures but he didn’t like them

✅✅
asking about his mental status according to house classification?
Hysterical

Houses classification*****

1- class1 ..... philosophical 👉best prognosis


2- class2..... Exacting 👉 lot of questions
3- class 3 ..... hysterical 👉 not accept the denture = worest prognosis

4- class 4 ..... Indifferent 👉 don't care


2-Pt just received her new CD came complaining of weird face shape and muscle
fatigue?!! >>
Increased ovd ✅✅
Unfamiliar with the cd

3-Dentist noticed air bubbles comes out in the palatal dam area when he presses

✅✅
the palate what to do ?! >>
Reline

😅
Reduce the palatal dam area
Do wax something

✅✅
4-Pics of nodular gingival enlargement findings are plasma cell infiltrate >>
plasma cell gingivitis
✅✅
5-Zone of flame that used with gold heating
1-reduced Zone
2-oxidized
3-cold work
✅✅
6-What is the more “caustic” ?! Material must be handled carefully ?!>>
Formocresol
mta
Calcium hydroxide

7-Pt lost all her posterior teeth including premolars What type of occlusion ull

✅✅
recorded >>
Balance occlusion
✳️✳️8-How to transfer the anterior teeth occlusion to the articulator > study
cast??

9-Canine replaced implant what occlusion ?


• Unilateral


• Bilateral
• Mutually protec
10-Pt with occlusal tooth wear want to do full mouth rehabilitation what articulator
to use >>
✅✅
fully adjusted
or semi adjusted

✳️✳️11-Pt with class II lost his post teeth in one said and the other side have
occlusal wear what to do ?! >> bite equilibrium of the side with teeth and do the
rpd according to it

✅✅
12- 2 years ol pt for extraction what LA to use ?!! >>
2% lidocaine
3% lidocaine
2% articaine
3% articaine
13-Pt with mobile tooth after receiving her full acrylic rpd with wrought wire

Tooth supported ✅✅
Tissue supported
Hybrid
‫**اسنانها صارت تتحرك بعد ما استخدمت الطقم‬
‫الطقم اكريلك بس و روت وير‬

14-Bacteria resistant to penicillin what to use ?! >>

✅✅
Penicillin vk
Amoxicillin + clavulanate
Ampicillin

15-What causes bacterial resistance to antibiotics?! >>

✅✅
Adaptation
Mutation
Dna changes

✅✅
16-Pic of Mid root small circular radiolucency asking what to do next ?! >>
Do rct
Take x-ray from another angulation

17-Pic of GP over extension and sealer with apical radiolucency and asking
about management >>
Surgical endo ✅✅
Trephination

18-Pt complain of tooth sensitivity or mobility can’t remember 😅 after receiving


her metal casted rpd what is the cause !?>>

The reciprocal arm engages with the tooth before the retentive arm

The retentive arm engages with the tooth before the reciprocal arm ✅✅

✳️✳️19-Pt lost # 44 45 46 , she didn’t know that the Rpd could extend to the
other side what to do ?! >>
Explain to her that it’s important for the retention

Do it only in one side

20-The most common drawback of tunnel preparation for furcation treatment ?!


Hypersensitivity
✅✅
Root caries
Recession
Mobility

21-Pt with 3-4 cm enlargement in the parotid area what to do ?!

✅✅
Incisional biopsy
Fine needle aspiration

22-What clamp to use for upper 6 !?>>


W4
W5

✅✅
W6
W8
✳️✳️23-Dr received special tray 3-4 mm shot is some areas what impression
materialto use ?!
Something sulfide
Putty pvs
Regular pvs
Green wax
✅✅
24-What happen when do rest sit before guiding plane?
A- insufficient size rest
B- insufficient size plane
C-fructure
25-Ceramic veneers gain retention by ?! >>
Micro mechanical by etching the veneer and the tooth ✅✅
Chemical by saline coupling agent

👇🏾👇🏾‫جاب نفس الصورة دي ويسأل عن العالج‬-26


✅✅
Corticosteroid
‫💕‬
‫🏾🙏‬
‫‪Dr.Afnan‬‬
‫اذكروني بدعوة‬
‫ مارس‬٢٨ ‫مناقشة‬

1-How many days need for local delivery agent (tetracyclin)?


A-3
B-7

✅✅
C-21
D- 10
2-When to use tissue conditional material?

Tissue conditioner materials are the temporary soft lining materials that are used
to treat the damaged mucosa, which is mostly caused by the trauma of the old

✅✅
denture of the patient.
Also, tissue conditioners can be used as a functional impression material
3-Patient continues close his lip in tip of suction
And he get little vacuum or pressure

✅✅
A-pressure inside patient more than suction
B-pressure inside patient less than suction
C-use large tip suction
4-Vinyls gloves allergy, consider which type of allergy?
A-I
B-II

✅✅
C-III
D-IV

5-Patient have pain in one side of his head and pain of tooth in same affected
side , scare from light?
A-cluster headache
B-migraine headache ✅✅

✅✅
6-What is the sign for tmj disorder without reduction?
A-crepitus
B-clicking while opening and closing
B-cleanching
✅✅
7-Pain of soft tissue and bone loss around implant?
A-periimplantitis
B-perimucositis

✅✅
8-What it the best pulpal test?
A-cold
B-heat
C-electrical pulpal test

✅✅
9-Patient came for bleaching , which important thing you should check ?
A-caries
B-incisal
C-occlusal

10-Patient came with sensitivity after home bleaching , how to reduce the
sinsitivity?

✅✅
A-Hydrogen peroxide
B-desensitized

11-Patient with came injury tongue but he was superficial and no pain or bleeding
, what you should do ?
A-one layer

✅✅
B-two layer
C-no treatment follow up

12-What is the most common area more prone to caries in elderly patient?
A-pti and fissure

✅✅
B-incidal
C-cervical
✳️✳️13-Which of the following root mostly fracture during post prep ?

Dr.Afnan 💕
‫اذكروني بدعوة‬
‫ مارس‬٣٠ ‫مناقشة‬

🛑* Blood circulation of the gingiva around implant


less✅✅
same as tooth ,

, moreee

🛑* Pt with history of trauma, now he has difficulty in moving tmj w facial


, ankylosis ✅✅
deformity

or osterarcoma ??

🛑Pt of extra oral Swelling on mand which tooth is the cause


molar ✅✅

🛑Type of bacteria we eliminate when we do scaling smth like this


Bacteria. The bulk of the microorganisms that form the biofilm are Streptococcus
mutans and other anaerobes, though the precise composition varies by location

✳️✳️✳️
in the mouth. Examples of such anaerobes include fusobacterium and
actinobacteria

🛑-1st pre molar small carious lesion w pt has attrition what is type of the
✳️✳️
restoration

🛑Type of bacteria we eliminate when we do scaling smth like this


✅✅
🛑* When we take biopsy of irritation fibroma what is the media used to transfer
biohazard bottle with formaline ✅✅
it to the lab

🛑* Pt take diuretic w plavix we do teeth preparation and we want to take


impression, smth about retraction cord or we stop the medication !?
aluminum chloride retraction cord for hypertension ✅✅
🛑* Avulsion of pedo patient, parent is not answering can we then !?
-start the ttt without concent because it is emergency case ✅✅
-take the content from the teacher

🛑 Sodium hypochlorite accident why?? Type of needle or


engaging the needle tightly ✅

🛑* Tooth with abscess coming out what to do


✳️✳️
🛑 Dentist touch the chair with his hands, when to disinfect?!
Directly ✅✅
, or at the end of the session
🛑 Max Denture fall down when talking and pt have gag reflex !?
Over extende buccal flanges and posterior palatal seal✅✅

🛑 Which type of child abuse we as dentists can know or recognize


physical abuse ✅✅

🛑 Positions of dentists hand inside the patient mouth✅

🛑* Obturation for open apex for adult ?


MTA ✅

🛑* Best way to remove gutta percha , h file, k file, reamer , heat !?


‫ ع حسب اذا حنشيلها عشان نعمل بوست بنستخدم‬reamer

✅✅
‫ اما لو عشان نعمل‬retreatment ‫ بنستخدم‬h file
🛑*if you want to do preparation sup-gingival what u will use !
Retraction cord✅✅
- or surgical retraction .. etc

🛑*the IANB will anesthetized which tooth and you will be able to extract it
without pain ?
1-molar
2-premolar and first molar

✅✅
3-molar premolar canine incisor
4-premolar
🛑pt come with burning sensation since 2 days !! What is the diagnosis 1-
geographic tongue✅✅
2- median reh. Glossitis
🛑Most comfortable location of The condyle !
✳️✳️
🛑- smoking during pregnancy can cause which syndrome ?
Cleft lip and palat✅✅
🛑If you want to take swap or biopsy form where do you take ! Center
Peripheral✅✅

🛑 After Vomiting type of disinfectant !?


in chair = follow manufacture instruction
in floor = intermediate
✅✅
🛑 Some doctors take the certificate without attending the course or lecture, this
veracity ✅✅
violation of what ??

🛑* medication we don’t give to pregnant woman!?


Drug with pregnant:
*Contraindicated*
Asprin with high dose
Ibuprofen
Diclophenac
*Indicated*
Acetaminophen is usually use
Asprin with low dose

‫االسبرين بجرعات عاليه وديكلوفين الصوديوم‬

👍
Tetra & nsaid too

✅✅
‫ و‬flgyl also ‫خالل االشهر الثالث األولى‬

🛑Chemotherapy, home many days b4 chemotherapy we can do extraction


5 days for maxilla

✅✅
7 days mandible

🛑* What to use for tracing the sinus tract!?


Gutta percha 25 or 30 ✅✅
🛑Recession - technique of brushing !?
modified stillman tech.✅✅

🛑* If the patient during examination say (( aaaaah )) and the half of the soft
side , Which nerve is affected ! Vagus✅✅
palate move and the other side not moved and the uvula more toward the right

, facial
, glossopharyngeal .

🛑different between cemented implant and serrated implant !


‫مم‬٧ ‫السمنتد تحتاج فراغ أكبر‬
‫واقل مقاومة‬

✅✅
‫مم‬٤ ‫التانية مقاومة ويكفيها‬
🛑* Pt come to your clinic after scaling and polishing have generalized gingival
ulcer and pain what you will do ?

✅✅
1-analgesic
2- apply steroid in the ulcer
3- apply topical LA in the ulcer
4- repeat the scaling and root planning

🛑time of neonatal teeth present


1-30 days ✅

🛑Class I malocclusion
✅✅
🛑Normal occlusion✅

🛑* Akinosy technique✅
🛑Implantsssssss and the spaces✅✅
🛑* Non working condyle position
In this case the non-working condyle is seen rotating downwards and medially.
During mandibular lateral excursions, the side of the mouth to which the


mandible has moved is known as the working side, while the side of the mouth
from which the mandible has moved away is known as the non-working side

🛑Curve of wilson✅✅
🛑risk of impacted canine
lateral root resorption ✅✅
🛑Phenotype
‫سؤال مشابه‬
‫السؤال كامل‬

35. Which of the following periodontal phenotypes is associated with thinner


mean of buccal bone thickness?
A .thin flat phenotype


B.thick flate phenotype
C.thin scalloped phnotype
D .thick scalloped phenotype
🛑Ortho head gear
read about it ✅✅

🛑pic about ludwidge angina !✅

🛑Wilson curve in prostho✅


🛑 S curve in prostho✅
‫💕‬
‫🏾🙏‬
‫‪Dr.Afnan‬‬
‫اذكروني بدعوة‬

You might also like